Söker efter Peter och E

 

59 frågor/svar hittade

Energi [334]

Fråga:
Hur många kärnkraftverk finns det runt Östersjön?
/

Svar:
Det finns två i Sverige; Forsmark och Oskarshamn, ett i Ryssland i St Petersburg och två i Ignalina, Litauen. Greifswald i norra Tyskland är nerlagt.

*

Kraft-Rörelse [952]

Fråga:
Om man tar ett glas med vatten och lägger ett papper ovanpå och sedan vänder detta så att papperet kommer neråt så blir papperet kvar. I några olika böcker har jag läst att det beror på att papperet buktas ner en aning pga vattnets tyngd. Den innestängda luften får då större plats, varför lufttrycket blir lägre inne i glaset än utanför. Nu har jag dock blivit varse om att detta skulle vara fel, och skulle därför vara tacksam om ni ville förklara hur det egentligen fungerar.
/Johan L, Högskolan i Kristianstad, Kristianstad

Svar:
Låt oss titta på de krafter som verkar på vattnet i det uppochnedvända glaset. Glaset påverkar med en kraft, sedan finns det tyngdkraften och luftens tryckkraft på pappret. Det är helt enkelt luftens tryck som trycker upp pappret och därmed vattnet. För att vattnet skall komma ur glaset måste luft komma in. Det går inte eftersom det våta pappret tätar bra mot glaset.

Försök: Gör försöket och se åt vilket håll pappret buktar.

Diskussion

Ovanstående fungerar även om glaset är vattenfyllt till hälften från början och då uppstår FRÅGAN; Varför fungerar det? Locket av tex papper, buktar uppåt, men det fungerar även med en aluminiumskiva, som inte buktar synligt uppåt. Dessutom, vad händer med det trycket som fanns (gavs) av den innestängda luften, när glaset vänds? Vi kan ju inte prata om "horror vacuui" i det här läget. Jag har alltså ett halvfyllt vattenglas, med säg 1 bars lufttryck i och utanför glaset, jag lägger på ett lock, samma lufttryck under som ovanpå locket. Vänd glaset, luftbubblan hamnar överst. Locket buktar inåt och sitter kvar, men rimligen borde jag ha 1 bars tryck från den innestängda luften plus ”vattenpelaren”, eller så borde ”vattenpelaren” sjunka nedåt och minska trycket lite ovanför. /Ole Lundgren

Ole! Ditt resonemang låter korrekt, men det är det inte. Lufttrycket i luftbubblan är inte detsamma, då kan man inte få jämvikt. Om man antar att vattenpelaren är 1 dm, så behöver vi ändra lufttrycket med bara 1% för att kompensera för vattenpelaren. Detta sker dels genom att pappret buktar ut lite eller att lite luft och/eller vatten läcker ut när man vänder glaset. Detta märks nästan inte eftersom det rör sig om en så liten korrektion.

Det finns också en annan effekt som kan hjälpa till med att hålla vattnet uppe: adhesion. Adhesion är klibbighet mellan två medier. Adhesion varierar mycket med mediernas egenskaper, men för vissa kombinationer kan adhesionen bidra.

Jag har utfört lite försök med en liten PET-flaska och ett plastlock. Adhesionen mellan plasterna är liten så denna effekt kan man bortse ifrån.

För det första kan man observera att ju mindre luft man har i flaskan, desto bättre hålls locket fast. Det är precis vad man väntar sig: mindre luftbubbla kräver en mindre volym för att ändra lufttrycket tillräckligt. Med 1/3 vatten och 2/3 luft är det svårt att få locket att fastna.

Slutligen stack jag hål i botten av flaskan. Vattnet rann då naturligtvis ut eftersom undertrycket i luftbubblan försvann. /Peter Ekström
/GO/lpe

Se även fråga 10835

Nyckelord: lufttryck [23];

*

Universum-Solen-Planeterna [1422]

Fråga:
Mina elever funderar över cepheider och vill veta mer. Vad är det för skillnad på ovanstående och pulsarer (neutronstjärnor )Utgångspunkt: Peter Nilssons artikel:"Rymdens och tidens gränser". Detta inom kursen naturkunskap, ämnesomr. Astronomi. Tacksam för svar innan terminens slut Hälsningar Lena Cronström
/Lena C, Tycho Brahe skolan , Helsingborg

Svar:
Cepheider är en typ av stjärna vars ljusstyrka varierar periodiskt. Under en expansionsfas ökar ljusstyrkan relativt långsamt. Sedan drar stjärnan snabbt ihop sig och ljusstyrkan avtar fort. Perioden för dessa "blinkningar" kan vara från några dagar till ett år, men varje stjärna har sin egen fasta periodtid.

Det intressanta med cepheiderna i motsats till andra stjärntyper vars ljusstyrka varierar är att det finns ett enkelt samband mellan periodtid och stjärnans ljusstyrka. Denna egenskap gör att cepheiderna används för att bestämma avståndet till närbelägna galaxer.

Fundera Hur går denna avståndsbestämning till?

Pulsarer däremot är neutronstjärnor som är mycket kompakta objekt och roterar mycket snabbt. De sänder ut radiovågor i vissa riktningar. Eftersom de snurrar så sänds dessa radiovågor ut likt ljuset från en roterande fyr.

Fundera Varför roterar en neutronstjärna fort. Ledning: Tänk på att de har bildats när en vanlig, långsamt roterande stjärna dör.

Beräkna En neutronstjärna har samma densitet som en atomkärna. Vilken diameter har en neutronstjärna med massan = 5 solmassor?


/GO

*

Materiens innersta-Atomer-Kärnor [1513]

Fråga:
Hejsan! Satt och tänkte på Einsteins berömda uttalande gällande kvantmekaniken, som han ju satte sig emot. Hade han ändå inte rätt i grund och botten då han sade att "Gud kastar inte tärning med oss"? För om jag förstått saken rätt så går hela kvantmekaniken ut på en gräns, från vilken vi inte kan fastställa både en partikels läge och dess hastighet exakt. Vi är oförmögna att göra det, men det behöver väl inte betyda att den inte har exakta värden på läge och hastighet?
/Nina , Norreport, Ystad

Svar:
Här kommer du in på mycket spännande och grundläggande frågor, svåra också. Det du beskriver kallas Heisenbergs obestämbarhetsrelation, och den har varit under diskussion med ungefär de resonemang som du tar upp. Alltså, det är en sak vad vi kan mäta, och en annan om det finns en "exakt" verklighet bakom.

Slumpmässigheten i kvantmekaniken är faktiskt mycket mer fundamental. Man har på senare år kommit underfund med, att alla delar av universum, i en viss mening, tycks stå i ögonblicklig kontakt med varandra. Den som kom på det hette John Bell.

Det där låter som rena nippran, inget kan ju gå fortare än ljuset! Det som inte kan transporteras snabbare än ljuset är energi och information. Vissa fysiker tror, att vi här kan få en djupare förståelse för slumpmässigheten i kvantmekaniken. Den tycks behövas för vi inte ska kunna skicka information ögonblickligen. Om det skulle ha varit möjligt, skulle universum ha sett helt annorlunda ut. Det här går nog inte att förklara så att du förstår allt. Kanske ingen kan förstå det riktigt.

Det finns ett annat exempel på denna gåtfulla fjärrkontakt, som varit känd långt före kvantmekanikens tillkomst: Machs princip. Det finns ingen absolut rörelse, men det finns absolut rotation. Sitter vi i en roterande ihålig kula och inte kan se ut, kan vi inte avgöra om den rör sig, men vi märker att kulan roterar. Vi känner av centrifugalkraften. Vi måste alltså på något vis stå i kontakt med något utanför.

I den engelska tidskriften New Scientist, 22 augusti 1998, finns en artikel, som tar upp dessa svåra, men intressanta frågor. Den har just titeln "Why God plays dice" eller, på svenska "Varför Gud kastar tärning" (se bilden nedan).

Tillägg 2/3/05 (Peter E):
Som Kaj säger är detta en mycket djup och svårförståelig aspekt av kvantmekaniken. Under länk 1 nedan finns länkar till Einstein-Podolsky-Rosen (EPR) artikeln, Bells artikel och en beskriving av Aspects experiment. Bell's Theorem är en relativt svår artikel som emellertid innehåller en länk till en förenklad analogi (länk 2 nedan). En varning dock igen: det är inte enkelt!

Sammanfattningsvis kan man säga att Aspects experiment (och flera senare) till 100% bekräftar kvantmekaniken. Vi får acceptera slumpmässigheten och det tycks inte vara något problem med relativitetsteorin eftersom man inte kan få fotonerna i försöket att överföra någon information.

Se även EPR_paradox .



/KS/lpe

Se även fråga 951

Nyckelord: EPR, Bell, Aspect [3];

1 http://www.drchinese.com/David/EPR_Bell_Aspect.htm
2 http://www.upscale.utoronto.ca/GeneralInterest/Harrison/BellsTheorem/Analogy.html

*

Universum-Solen-Planeterna [2371]

Fråga:
Varför har det inte startats något projekt att skicka en människa till Mars? Det finns säkert många som skulle ta riskerna för att vara första männsikan på Mars.
/Peter S, Sollentuna

Svar:
Hej Peter! Att skicka en människa till Mars medför två stora problem, jämfört med obemannade prober. För det första, resan tar i storleksordningen år (1-2), och det krävs stora resurser för att hålla en människa levande under en så lång tid (och kanske också underhålla hennes förstånd!). Det andra problemet är att man av etiska skäl antagligen vill ta hem henne igen! Det krävs också att stora mängder bränsle transporteras till Mars, så att den bemannade proben kan lyfta från Mars och komma loss från planetens gravitation.

Det är svårt att tänka sig att en människa skulle kunna utföra bättre eller mera utförliga mätningar och observationer än vad en avancerad robot kan. Den sistnämnda kräver ju mycket lite underhåll under resan (den kan till och med stängas av!) och behöver inte tas hem igen. Den kan också arbeta på Mars under flera år, om dess energiförsörjning sker genom solceller. Det är därför inte så konstigt att man satsar på obemannade rymdfärder istället!

NASA har också numera en allmän policy att man satsar på mindre och flera "missions", istället för få och storslagna. Om inte annat minskar det dramatiken vid ett misslyckande! Naturligvis drömmer vissa entusiaster om att placera en människa på Mars, men jag är övertygad att om det sker så är det mera av politiska och strategiska skäl, än vetenskaplig nyfikenhet på vår röda granne!
/Tomas Brage

Nyckelord: rymdfärder [23]; Mars [12];

*

[2377]

Fråga:
1. är det kul att vara fysiker?
2. hur lång tid tar det att bli en bra fysiker?
3. vad gör man när man är fysiker?
4. vad heter du?
5. hur länge har du varit fysiker?


/Sofia S, ONSALA

Svar:
Tack, Sofia, för det trevliga frågorna!

  1. Ja, jättekul! Arbetet är mycket omväxlande, och man förstår mycket av världen omkring sig (dvs den del som styrs av naturlagar)
  2. Att bli en bra fysiker är svårt att säga. För att bli en bra fysiker behöver man en viss talang. Om man säger att man blir fysiker när man har en doktorsexamen i fysik, så är det normalt universitetsstudier på 4-5 år för en grundexamen (filosofie magister eller civilingenjörsexamen) och ytterligare 4 år för att bli doktor.
  3. Mycket varierande. Man kan t.ex. bli lärare på gymnasium, lärare/forskare på universitet, forskare (ofta projektledare) i industrin, byråkrat (bra byråkrater behövs också!). Eftersom fysiker sysslar mycket med datorer - du vet väl att World Wide Web uppfanns av en fysiker Tim Berners-Lee - så går många till datorindustrin
  4. Peter Ekström
  5. Enligt min ovanstående definition, 22 år (oj vad tiden går!)

Hoppas detta kan inspirera dig att bli fysiker - vi behöver fler tjejer!
/Peter Ekström

*

Blandat [2408]

Fråga:
Idag är kvantmekanik (kvantfysik), kaosteorier med flera väldigt "inne". Framför allt tillämpas dessa inom "icke-fysiska" områden som till exempel psykologi. Ett exempel på detta är Danah Zohar och hennes böcker "Kvantjaget" och "Kvanttänkandet". I vilken utsträckning har fysiska institutioner i Sverige, som er, relationer med andra ämnesområden? Hur ser dessa eventuella relationer ut?
/Clas M, Göteborg

Svar:
Intressant fråga.

Våra relationer med andra ämnen rör främst teknik och medicin. Jag själv som forskar inom kaosteorin har samarbete med kardiologer. Vi försöker hitta matematiska modeller för att beskriva hjärtats dynamik.

Det enda projekt av den typ Du nämner som jag känner till är samarbete mellan fysik och filosofi i Göteborg som förekom för ca 10 år sedan.

Kommentar från Peter Ekström:
De exempel du nämner (jag känner själv till en golfinstruktionsbok som heter Kvantgolf) är för att med en väl etablerad teori (som dessutom få förstår) öka trovärdigheten och statusen för ens egna teorier. Dessa exempel har alltså inget med kvantmekanik att göra, och 'kvant' lånas bara för att kvantmekaniken är en utomordentligt framgångsrik teori. Detta missbruk av en etablerad teori för att ge ens egna funderingar större vikt är för mig en tydlig varning för pseudovetenskap .

Det finns dock, som Gunnar Ohlén säger, fortfarande mycket allvarligt syftande funderingar om man i någon mening kan "förstå" kvantmekaniken.
/GO

Nyckelord: pseudovetenskap [11];

*

  [2957]

Fråga:
Hej på er. Jag har en fråga som jag har funderat på i många år som jag nu hoppas att ni kan hjälpa mig med. 'Om ett tåg kör i 100 km/h i en riktning och man kör motorcykel i tåget åt andra hållet i 100 km/h så står man still om man jämför med verkligheten utanför. Vad händer om tåget kör in i en bergvägg och går från 100 till 0 km/h direkt och man samtidigt frikopplar motorcykeln ?' Flyger man bakåt ? Framåt ? Står man still ? Snälla förklara detta för mig. MVH / Peter Johansson
/Peter J, Lerums gymnasium, Floda

Svar:
Ja, man står still. Man kör av sista vagnen innan den nått bergväggen, och landar stillastående på marken. 
/KS

*

Blandat [4662]

Fråga:
Hej, jag skulle vilja veta hur man tillverkar krut.
/patrik O, jacob skolan, hässleholm

Svar:
Slå på krut i Nationalencyklopedin ! Det klassiska svartkrutet består ju av svavel, kol och salpeter. Hur fick man fram dessa produkter förr? På 1600-talet till exempel.

Kol var lätt att få tag på. Vanligt träkol duger bra.

För att få fram svavel hettade man upp svavelkis utan lufttillträde. En del svavel förångas och fastnar (sublimerar) på kallare ytor, så kallad svavelblomma.

Salpeter var knepigare att få fram. I ladugårdarna rann urin och andra organiska substanser ner i marken. Vi känner nu till att vissa bakterier får sin energi genom att oxidera olika kväveföreningar till nitrat. Med kanske 10 års mellarum kom kungens män, och tvingade bonden att gräva upp ladugårdsgolvet. Sedan följde en omständlig process. Man lakade ut jorden på salter och genom flera olika steg renades lösningen. Det sista steget, salpetersjudningen, innebar att man upprepade gånger kokade upp och kylde ner kitteln med saltlösningen. Slutprodukten var två fraktioner, som huvudsakligen bestod av natriumklorid (koksalt) och kaliumnitrat (kalisalpeter). Salpetersjudaren kunde ingen kemi (i vår mening), men han visste hur han skulle göra.
/KS/lpe

*

Universum-Solen-Planeterna [4924]

Fråga:
Hej! Vi undrar - vad består en asteroid av och hur blir den till?

Jonas, Sebastian, Andreas,Miriam, Micael och Peter
/Jonas N, Tingbydal, Kalmar

Svar:
Asteroiderna besår av sten, mer eller mindre porös. De kom till mycket tidigt i solsystemets historia. Man tror att jorden och de andra planeterna kom till genom att asteroidliknande himlakroppar slog sig samman. Att somliga blev kvar, beror antagligen på påverkan från Jupiter.

Rymdsonden NEAR gick i början på år 2000 in i en satellitbana kring satelliten 433 Eros. Kolla länken!
/KS

1 http://antwrp.gsfc.nasa.gov/apod/ap000308.html

*

Universum-Solen-Planeterna [7332]

Fråga:
Efter som astronomisidan ligger ner så tänkte jag att ni kanske kan besvara min fråga. Jag läser till Ma/NO lärare 4-9 och det har kommit upp en del undringar angående det faktum att man från vår måne kan se den kinesiska muren utan några som helst hjälpmedel. Jag undrar om detta stämmer eller det bara är en skröna som är svår att överbevisa! Mvh Christel
/Christel G, Lut, Malmö

Svar:
Det är definitivt en skröna. De minsta föremål man kan se med blotta ögat är flera hundra kilometer.

Tillägg av Peter E:

Sedan är det en annan sak att man kanske kan se den indirekt eftersom den utgör en landsgräns. Detta kan medföra att man har olika odlingar på ömse sidor om gränsen. Denna skillnad vet jag att man kan se på vissa statsgränser i USA på fotografier från satelliter. Nedanstående bild (från NASA, länk 1) visar ett extremt exempel på detta från en nationalpark i Tunisien. Bildtexten:

Sidi Toui National Park, in the southern half of Tunisia, close to the Libyan border, viewed by the Enhanced Thematic Mapper Plus (ETM+) instrument on NASA's Landsat 7 satellite on December 13, 1999. Native vegetation can be seen returning inside the borders of this protected park (approx. 7 kilometers wide), established in 1993 to protect the region against desertification. The effects of continued agriculture, overgrazing and drought can be seen on the surrounding arid landscape. (NASA/Jesse Allen/Landsat,USGS)


/KS/Peter E

Se även fråga 3699

1 http://www.boston.com/bigpicture/2009/01/earth_observed.html

*

Partiklar [7548]

Fråga:

1) På vilket sätt ger Higgspartikeln uphov till det som vi uppfattar som massa?

2) Om man inte hittar partikeln, så säger man att hela Standardmodellen rasar, vad menar man med det?

3) Hur tror ni att man kommer se Higgs i den nya LHC:n (Large Hydrone Colider), som man håller på att bygga på CERN i Schweiz?

4) Vem uppfann Higgs??Jag antar att det var en man som hette Higgs??..eller?
/Danne F, Katedralskolan, Uppsala

Svar:

1. Sajt 1 försöker förklara det på enkel svenska. Sajt 2 är på engelska.

2. Då måste vi hitta något annan mekanism som genererar partikelmassor.

3. Higgspartikeln väntas mest sönderfalla till 2 b-kvarkar, som vardera vanligen sönderfaller till två strålar av hadroner (starkt växelverkande prtiklar). Man kommer leta efter kollisioner där det finns 4 hadronstrålar.

4. Peter Higgs är engelsman. Han var inte alldeles ensam om detta, så är det för det mesta.
/KS/lpe

Se även fråga 1375

Nyckelord: higgspartikeln [10]; standardmodellen [24];

1 http://courses.physics.kth.se/5A1310/elementar/symmetrier.html
2 http://atlas.web.cern.ch/Atlas/documentation/EDUC/physics9.html

*

Kraft-Rörelse [7676]

Fråga:
Hej! Det är en sak jag inte förstår med Einstein och hans relativitets teori: Ljusets hastighet är ju absolut, det är lika för alla oberoende om man är i rörelse eller ej, men då borde inte dopplereffekten gälla ljuset! Jag ska förklara lite hur jag tänker.

Jag har läst en bok som heter "Einstein för nybörjare" av Joseph Schwatz & Michael McGuinness. I den förklarar de väldigt bra hur det kan uppfattas att ljusets hastighet är absolut. Det beskrivs ungefär så här: Tänk dig ett tåg med en vagn i rörelse. I mitten av vagnen finns det en lampa och en man (Peter). Det finns en dörr i de båda ändarna av vagnen som vi låtsas kan öppnas av ljuset från lampan. Det finns även en man på marken utanför tåget (Anders). När Peter tänder lampan tycker han att dörrarna öppnas exakt samtidigt. Men Anders tycker att den bakre dörren öppnas aningen innan den främre dörren. Det gör han eftersom att ljuset från lampan rör sig med hastigheten c i förhållande till honom och den bakre dörren möter ljuset. Alltså öppnas den bakre dörren lite innan den främre.

Sen har jag också börjat läsa en bok som heter "Einsteins universum" av Nigel Calder. I den står det om dopplereffekten. När jag läser i den tycker jag inte att det stämmer med vad jag vet (tror att jag vet). Det står ungefär så här: "Om du sitter i en rymdraket och åker mot solen fortare och fortare kommer du tycka att solen blir blåare och blåare eftersom ljusets våglängder trycks ihop när du möter det. Kortare våglängd = energirikare ljus." Men ljusets hastighet beror ju på hur fort jag själv rör mig! Våglängderna borde inte kunna tryckas ihop. Jag förstår inte hur det fungerar!

Jag skulle också bli glad om ni kunde förklara för mig på något bra sätt varför längden på t.ex. den ovan nämda tågvagnen är relativ för Peter och för Anders.

Tack!
/Martin L, Långbergsskolan, Nyköping

Svar:
En orsak till att dessa saker är svåra att förstå, är att relativitetsteorin inte behövts i vardagslivet. Våra hjärnor har inte utvecklats så att vi ska ha det lätt med relativitetsteorin (eller kvantfysiken).

Kommentar till en av de sista meningarna: Ljushastigheten beror inte på hur fort du rör dig. Det är just utgångspunkten för relativitetsteorin. Det här var inget Einstein hittade på, det var experimentella resultat. Det stred mot Newtons mekanik, och Einstein insåg att den måste ersättas med något annat.

I stället för att kommentera de tankeexperiment du presenterar, vill vi hänvisa till en annan källa: Nationalencyklopedin . Artikeln där är mycket bra. Tyvärr har det insmugit sig ett par fel i figur 1, som gör den obegriplig. Observatören i rymdskeppet ska ha beteckningen B (har ingen beteckning nu). Den vänstra observatören på marken har beteckningen A (rätt), men den högra observatören har beteckningen B (fel, ska vara A). I ekvationen längst ner ska v ersättas med v2. Information om detta finns både i rättelseblad och supplementband. Tro nu inte att man kommer in i relativitetsteorin bara genom att läsa igenom artikeln. Man får nog ägna åtskilliga timmar åt det.
/KS

*

Elektricitet-Magnetism [8878]

Fråga:
Vad är Rogets spiral för något. Hur funkar den?
/Lena S

Svar:
Det var lite knepigt att hitta något om Rogets spiral, men nu vet vi. Peter Mark Roget (1779 - 1869) var engelsman och han uppfann en apparat som i viss mån påminner om en elektromekanisk ringklocka.

Man hänger upp en ganska lös metallspiral i ett stativ. Spiralens nederände slutar med en spets, som doppar ner i en skål med kvicksilver. Nu ansluter man ett batteri, ena polen till kvicksilvret, den andra till spiralens överände. Då går det en ström genom spiralen. Ledare med strömmen åt samma håll dras mot varandra. Det får spiralen att dra ihop sig, så att spetsen kommer upp ur kvicksilvret. Då bryts strömmen, och spiralen tänjs ut så att spetsen åter når kvicksilvret. Spiralen dras åter ihop, och så vidare.

Vad man ser är en spiral, som hela tiden hoppar upp och ner, så länge batteriet varar. Troligen en helt onyttig grej, men säkert en kul demonstration.
/KS

*

Blandat [9841]

Fråga:
Är tiden kontinuerlig eller diskret(kvantiserad eller vad man ska kalla det)?

Mvh Peter
/Peter K, Malmö

Svar:
Man räknar i allmänhet med att tiden är kontinuerlig, men visst förekommer det funderingar i den andra riktningen.
/KS

Se även fråga 6672

*

Värme [10577]

Fråga:
Hej! Jag har hört att det ska finnas "något" som inte ändrar temperatur. kan det verkligen stämma?

Tacksam för svar/Peter
/Peter W, Komvux, SUNDSVALL

Svar:
Man kommer att tänka på fasövergång. Där ändras energitillståndet vid konstant temperatur. När man kokar vatten, tillför man energi, men temperaturen ändras inte. Smälter man is i ett glas (och rör om) är temperaturen 0 oC ända tills all is har smält.
/KS

Se även fråga 5083 och fråga 2737

*

Blandat [12746]

Fråga:
Jag och mina vänner genomförde "anden i glaset" som går ut på att ställa frågor som sedan en "ande" ska svara på. Man skriver alla bokstäver i alfabetet och "ja" och "nej" på ett stort papper eller en skiva. Man värmer ett glas över ett ljus och viskar sedan sin fråga i glaset. Glaset ställs på underlaget, alla lägger ett lätt finger ovanpå och därefter glider glaset runt och svarar på frågan. Jag trodde inte på det här alls innan vi gjorde detta men alltför många gånger fick vi svar vi inte kunde förklara. Många gånger kunde en eller två av oss (vi var tre stycken) lyfta på fingrarna och glaset gled runt ändå. Styrs glaset av våra fingrar fast man är övertygad om att man håller så lätt man kan och inte påverkar glasets riktning? Jag frågade efter mina fyra sista siffror i personnumret och det svar jag fick var rätt. Ingen av de andra kunde dessa siffror. Vi frågade många saker om den närmsta framtiden och kusligt nog har det också stämt. Hur fungerar det?! Jag blir tokig av att fundera på det!
/Christin B, Högskolan i Trollhättan, Trollhättan

Svar:
Christin! "Anden i glaset" och liknande fenomen har inget med något övernaturligt att göra. I stället styrs glaset av deltagarna själva eller (om de inte alls berör det) av "slumpen". (Om glaset är varmt och ytan det vilar på är litet "fet" är friktionen liten och då kan glaset glida runt till synes utan yttre påverkan.) Flera undersökningar har visat att ditt antagande om att glaset styrts av era fingrar, trots att ni var fast övertygade om att ni inte påverkade det, är helt riktigt.

För att testa detta, om du nu har lust att göra om "andeleken" igen, pröva följande: tänk ut ett tal eller ännu hellre ett nonsensord (qwo56ngkk eller liknande), men berätta det inte för dina vänner. Fråga "anden i glaset" om vad du tänkt ut, men låt bara dina kompisar hålla sina fingrar på glaset. Jag tror inte att du kommer att bli förvånad över resultatet.

Man kan också göra ett annat litet test för att pröva hur starkt ens "undermedvetna" är: fäst ett gem i en bit sytråd. Ta tag i den fria sytrådsändan med tumme och pekfinger på en hand, sätt ner armbågen i bordet och låt gemet hänga fritt någon centimeter ovanför bordet. Koncentrera dig nu mentalt och försök få gemet att börja röra sig som en pendel med enbart "tankekraft". I de allra flesta fall börjar gemet efter en stund att röra sig! Det visar sig att hjärnan sänder ut pulser till handens muskler så att de gör nästan omärkliga rörelser för att åstadkomma det man vill ska hända!

Kommentar till det sista: jag har gjort detta experiment med en hel grupp. Alla fick gemet att svänga utom jag för jag var övertygad att det INTE skulle svänga! /Peter E
/Margareta H

Nyckelord: parapsykologi [6];

*

Blandat [12406]

Fråga:
Jag skulle vilja veta mera om magnetkamera och dess uppbyggnad. Varför är uppfinningen så betydelsefull att Nobelpriset delas ut till Paul C Lauterbur och Peter Mansfield för deras upptäckter?
/Malin P

Svar:
Börja med att läsa artikeln om magnetisk resonanstomografi i Nationalencyklopedin . Sedan finns mycket material på Nobelstiftelsens hemsida .
/Peter E

*

Universum-Solen-Planeterna [12602]

Fråga:
När började den moderna kosmologin?
/Veckans fråga

Ursprunglig fråga:
Hej, jag har ett specialarbete om moderna kosmologin, när började den?
/Mohammad H, Brännkyrkagynasium, Midsommarkransen

Svar:
Hej Mohammad! Det är omöjligt att svara på eftersom begreppet 'modern kosmologi' inte är väldefinierat. Man kan säga på 1600-talet då en korrekt världsbild började växa fram. Fler milstolpar: Einsteins allmänna relativitetsteori 1916; upptäckten av Hubble att universum expanderar (1929); Gamows formulering av Big Bang teorin (40-talet); förståelsen i början av 50-talet hur sjärnorna producerade sin energi och hur grundämnena bildats; Penzias och Wilsons upptäckt 1965 av den kosmiska mikrovågsstrålningen; HST (HubbleSite ); de senaste resultaten från Wilkinson Microwave Anisotropy Probe (WMAP) som ger mycket noggrann information om universum, bland annat vad universum består av, se bilden nedan från "NASA/WMAP Science Team".

Börja med att slå på kosmologi i Nationalencyklopedin , där finns en bra artikel av Peter Nilsson. Wikipedia har ett par bra artiklar som mycket detaljerat behandlar även historiska aspekter: Cosmology och Timeline_of_cosmology . Sedan finns det många källor att fortsätta med, ovanstående och länken nedan ger en början.



/Peter E

Nyckelord: kosmologi [33];

*

Ljud-Ljus-Vågor [12816]

Fråga:
Hej! Vi skall jobba med optik de närmsta veckorna. Som en introduktion tänkte vi visa att vitt ljus kan delas upp i färger. Hur skall vi montera vårt materiel så att vi får ett stort och fint spektrum , tex på väggen. Alltså ett "recept" på hur vi skall montera linser etc. för att få det så bra som möjligt. Tack på förhand.
/Andreas H, Kärralund, Göteborg

Svar:
Andreas! En aspekt som gör fysik så roligt är just den enorma möjlighet som finns att även med enkla medel göra egna försök och därmed kunna illustrera och undersöka olika fysikaliska grundprinciper och sammanhang. Det är naturligtvis viktigt att demonstrationer blir "så bra som möjligt", men håller du inte med om att själva arbetet med att - som i ditt fall - åstadkomma ett fint spektrum på väggen är en del i ditt och dina kamraters egna lärande om optik?

Vi föreslår att du och dina kamrater försöker er på att repetera Newtons prismexperiment - läs mer om principerna bakom detta under länk 1. Ni behöver en "vit" ljuskälla, minst en smal spalt och två prismor. Börja med att montera alla delar så att de har samma höjd över bordet, och ta sedan ett steg i taget och pröva ut vilket avstånd till er "projektionsyta" (väggen) som blir bäst när ni har först ett prisma i ljusstrålen - ljuset böjs ju av, så det får ni ta hänsyn till! Introducera sedan ytterligare ett prisma och se hur ljuset "böjs ihop igen".

Jag tror ni kommer att märka att det är både intressantare och roligare att själva "leka fram" de bästa försöksbetingelserna, än att slaviskt följa instruktioner från andra om avstånd etc. Genom att arbeta på detta sätt kommer ni mycket närmare hur "riktig forskning" går till än ni kanske anar!


Nyckelord: prisma [1]; spektrum [11];

1 http://micro.magnet.fsu.edu/primer/java/scienceopticsu/newton/

*

Värme [13206]

Fråga:
Hej fysiker! Om jag stoppar ner en isbit i kyld (-20 grader) t-röd, och ställer in det i frysen och väntar ett tag så "försvinner" isbiten. Vad är det som händer, smälter den eller löser etanolen upp den på något vis. Tacksam för svar.
/Peter C, Örebro

Svar:
Hej Peter! Isbiten smälter och vattnet blandar sig med t-röd som har låg fryspunkt. Förutsättningen är att isbiten är liten jämfört med rödspriten, annars skulle utspädningen höja fryspunkten.
/Peter E

*

Kraft-Rörelse [14380]

Fråga:
Om man till en massa (1 kg) som har hastigheten (10 m/s) tillför (150 J) kinetisk energi, kommer hastigheten att öka till (20 m/s). (e = 1/2 * m * v*v). När massan har hastigheten 10 m/s har den en kinetisk energi på (50 J). D.v.s. Det krävs 3 gånger mer energi att accelerera massan från (10 m/s ---> 20 m/s) än det krävs att accelerera massan från (0 m/s ---> 10 m/s).

Betyder detta att om man sitter i en lastbil som kör 10 m/s och i den har en massa på 1 kg som man inne i lastbilen accelererar till en hastighet (relativt med lastbilen) 10 m/s i färdriktning d.v.s 20 m/s absolut hastighet, behöver 3 gånger mera kinetisk energi än om man accelererar massan när lastbilen står stilla?

Detta är något som jag inte får att riktigt hänga ihop! Några upplysningar om var jag tänker fel.
/Anton M, Stockholm

Svar:
Anton/Peter! Din fråga är mycket bra - som du beskriver problemet verkar det märkligt. Det korta svaret är att observatörer i olika inertialsystem (icke accelererande system, se fråga 19113 ) inte är överens om utfört arbete. Låt oss försöka reda ut problemet ordentligt!

Kinetiska energin för en kula på 1 kg och hastighet 10 m/s är som du säger 50 J och kinetiska energin vid en hastighet av 20 m/s är 200 J. Skillnaden är alltså 150 J.

Vi har två system: marken och lastbilen. Deras relativa hastighet är konstant 10 m/s.

Du sitter på lastbilen och påverkar kulan med en konstant kraft F under en tid T tills den rör sig med hastgheten 10 m/s i lastbilens framåtriktning. Kulan har då den kinetiska energin 50 J sett från lastbilen.

Sett från marken är dess kinetiska energi 200 J efter din acceleration av massan, medan den före accelerationen hade kinetiska energin 50 J.

Sett från lastbilen får alltså kulan en energiökning av 50 J. Sett från marken är ökningen 200-50 = 150 J! Hur kan detta komma sig? Är inte energin bevarad? Kan vi använda detta för att göra en evighetsmaskin?

Nej, tyvärr inte. Om man räknar korrekt så är det inget problem. Observatörer i de båda systemen är överens om kraftens storlek F (den kan t.ex. visas med en kraftmätare) och de är överens om hur länge kraften appliceras T (vi talar här om normala hastigheter, vid relativistiska hastigheter har vi problem här).

Lastbilssystemet: tillförda energin är kraften*sträckan dvs Fx'. Medelhastigheten är (0+10)/2 = 5 m/s (likformig acceleration). Sträckan kraften verkar blir då x' = 5T, och den tillförda energin 5FT.

Marksystemet: tillförda energin är kraften*sträckan dvs Fx. Medelhastigheten är (10+20)/2 = 15 m/s (likformig acceleration). Sträckan kraften verkar blir då x = 15T, och den tillförda energin 15FT.

Vi ser att tack vare att sträckan x uppfattas som längre i det fixa systemet, så blir det tillförda energin tre gånger så hög i detta system, helt i enlighet med ovanstående värden 50 J och 150 J.
/Peter E

Nyckelord: inertialsystem [6]; rörelseenergi [14]; arbete [24];

*

Kraft-Rörelse [14410]

Fråga:
Hej Har funderat på en sak angående lufttryck. Om jag har försått saken rätt utöver luften ovanför mig ett ganska stor tryck på mig. Turligt nog trycker luften och vätskorna i mig tillbaka så jag inte blir krossad. Men när man skapar vakum, hur kommer det sig att behållaren inte imploderar. Man har ju sett dels gräddbullen som växer i en glasbehållare och även ett längre rör där en fjäder och mutter faller lika fort. Har inte kännts som om dessa behållare, särskillt inte röret, skulle kunna stå emot ett sådant tryck. Tacksam för svar.
/Peter B, Gullstrand, Landskrona

Svar:
Hej Peter! Behållaren imploderar visst om väggarna inte är tillräckligt starka. Normalt lufttryck är i runda svängar 105 Pa (=N/m2). Det blir 10 N/cm2. Trycket på varje cm2 är alltså 10 N vilket motsvarar tyngdkraften på ett kg. Annorlunda uttryckt väger luftpelaren på varje cm2 1 kg. Det fordras alltså en ganska stark behållare om den inte skall implodera!
/Peter E

Nyckelord: lufttryck [23];

*

Kraft-Rörelse [14459]

Fråga:
Hej! Bara en kommentar till 14455/13280.Jag försökte att leta själv bland tidigare svar, men förbisåg 'avancerad sökning', och att gå på namnet fungerade inte för mig i standardsökningen!

Det har tydligen blivit en hel del frågor(30) med tiden och jag är ytterligt tacksam för frågelådan, som besöks även för att se vad andra frågar om och får svar på!

Men den senaste frågan föranleddes av att det visat sig att (den nu nedlagda) tidskriften Elementa ställt frågan för många år sedan, och visade sig ha svaret attraktion(!!) mellan de två bubblorna i den vattenfyllda rymden. Det fanns således anledning för mig att försöka få tag på 13280 igen. (Det visade sig att frågan om den punkterade burken, som jag ställde rätt nyligen också finns i en gammal Elementa! Kanske borde jag ta mig igenom tidskriftsserien, men hur får man tid till det; den startade väl på 1920-talet!)
/Thomas Å, Märsta

Svar:
Hej Thomas! Dina många frågor är inget problem - de flesta är mycket bra och intressanta. Jag hoppas våra svar inte avviker alltför mycket från Elementas !

Elementas svar på bubbelfrågan (13280) är säkert fel. Förbindelselinjen mellan bubblorna har en brist på materia, så nettokraften på båda bubblorna måste vara bort från den andra bubblan.

För den punkterade burken (14451) säger du inte vad Elementa säger, men jag är ganska säker jag har rätt. Tricket är att inte bry sig om vad som händer när luften pyser in: titta bara på rörelemängden före och efter!

Kommentarer:

Elementas svar till pysande burken är samma, den rör sig ej. Texten lyder: "Burken kommer att ligga stilla. Gaserna kommer visserligen att påverka burken med en reaktionskraft, men denna kompenseras precis av tryckkraften, som gaserna utövar, när de träffar insidan av burken mitt emot hålet." Men om nu den inströmmande gasen erhåller en positiv rörelsemängd var finns motsvarande negativa (så att delta-p = 0)? /Thomas Ålander

Som jag sa: bättre att titta på slut- och begynnelsetillstånden! Vad som menas (och det är krångligt att kvantifiera) är att först får man en reaktionskraft bakåt men när gasen kommer till den motsatta väggen ett tryck framåt. /Peter E
/Peter E

Se även fråga 13280 och fråga 14451

*

Ljud-Ljus-Vågor [15146]

Fråga:
Hej hej! Jag undrar följande: Varför blir skuggan skarpare om man har en liten ljuskälla än en stor?

Man kan t ex ta sin knytnäve och börja med att hålla den framför en lampa och observera skuggan på vägen. Då är den suddig. För man doc knytnäven längre bort från vägen blir skuggan mindre men skrapare.

Varför?
/Peter L, Gärdeskolan, Stockholm

Svar:
Hej, hej Peter! Om ljuskällan är mycket liten har man liten halvskugga, om ljuskällan är stor har man stor halvskugga. Tänk att du befinner dig på gränsen mellan ljus och skugga. Ljuskällan är stor och du ser hela ljuskällan. Rör dig mot skuggan, så ser du att en del av ljuskällan är skymd. Mer och mer av källan kommer att försvinna och belysningen minskar. Om ljuskällan är liten (punktformig) försvinner källan omedelbart och du får en skarp skugga.
/Peter E

*

Värme [15744]

Fråga:
Uppvärmning i en mikrovågsugn
/Veckans fråga

Ursprunglig fråga:
Varför blir vissa muggar så varma i mikron fast mjölken är kall när jag värmer oboj
/Gabriel A, Grind, Norrtälje

Svar:
Gabriel! Är verkligen mjölken kall och muggen varm? Det låter mycket konstigt! Den normala förklaringen av mikrovågsugnens funktion är att energin från mikrovågorna (med c:a 10 cm våglängd) absorberas av vattenmolekyler. Det är alltså mjölken som värms. Muggen innehåller knappast vatten, så uppvärmingen av denna sker genom ledning från den varma mjölken.

Den enda förklaring till din observation som jag kan komma på är att muggen är av något material som kan absorbera mikrovågor. Kan lergodset t.ex. innehålla tillräckligt mycket vatten.

För att se om detta är korrekt kan du göra följande experiment (tillsammans med en vuxen för mikrovågsugnar kan ibland vara lite farliga): Ställ en tom mugg och en mugg med vatten i mikron. (Muggen med vatten är till för att ta hand om mikrovågorna - det är inte bra att köra mikron utan något att värma.) Kör en halv minut på full effekt. Öppna dörren och känn på muggarna. Vilken är varmast? Jag tror att den med vatten är varmast och den tomma muggen är kall. Skriv gärna en fråga till med resultatet av experimentet.

Tillägg från Gabriel 28/5:

Gabriel och jag (mamma) testade det du sa och stoppade en tom mugg i mikro och en med vatten. Den med vatten blev varmast men när jag tog den muggen som jag tycker blir så varm så blev den också varm men inte riktig lika varm som den med vatten i. När vi värmde ett glas blev det också varmt fast det inte var vatten i, varför blev det de? Den muggen som blir så himla varm är av lerkeramik.

Tillägg från Peter E:

Tack för inlägget med resultatet av fortsatta experiment!

Era observationer med en mugg med och en utan vatten (samtidigt!) är som man väntar sig. Trots att muggen med vatten kräver mer energi för att bli varm (både muggen och vattnet skall ju värmas och vatten kräver mycket energi) så blir den betydligt varmare än den tomma muggen. Den tomma muggen värms upp lite grann trots att den innehåller lite vatten (glas inneåller inget vatten alls).

Förklaringen är alltså att vatten absorberar mikrovågsstålning mycket bra men keramik och glas absorberar vatten dåligt. Därför kommer vattnet i muggen att värmas mycket, och muggen med vatten blir varm genom värmeledning från vattnet. Det blir emellertid inte mycket mikrovågor över för att värma den tomma muggen.

Om man emellertid ställer in bara en tom mugg, så måste ändå mikrovågorna ta vägen någonstans! De kommer att så småningom absorberas av muggen och väggarna i mikrovågsugnen. Väggarna och muggen kommer alltså i detta fallet att bli varma. Det är därför man rekommenderar att man aldrig skall köra mikrovågsugnen utan mat eller vätska.

Jag upprepade ert försök dels med en tom och en full mugg. Den tomma muggen blev bara ljummen medan den fulla blev varm. Ett ensamt tomt glas blir, precis som du säger, mycket varmt på kort tid. Men, igen, gör inte det försöket för mamma blir nog irriterad om hon blir av med mikrovågsugnen .
/Peter E

Nyckelord: mikrovågsugn [25];

1 http://www.explainthatstuff.com/microwaveovens.html

*

Materiens innersta-Atomer-Kärnor [15892]

Fråga:
Hej Peter I partikelacceleratorn LHC i Cern accelereras bl.a PB(+82), alltså blykärnan utan elektroner. Hur gör man för att "slita bort" samtliga elektroner?
/marcus e, Alléskolan, Hallsberg

Svar:
Hej Marcus! Man låter föraccelererade joner passera ett folie. Foliet "strippar" bort elektroner. Sedan väljer man med ett magnetfält ut de joner som är helt strippade.
/Peter E

*

Blandat [15926]

Fråga:
På en mikrovågsugn är det möjligt att ställa in olika typer av program t.ex. grill. Detta borde rimligtvis handla om att man ändrar våglängden på mikrovågorna (frekvensen). Varför gör man i så fall det? Hur fungerar det?
/Daniel D

Svar:
Daniel! Nej, grillen har inget att göra med mikrovågorna. Vissa större modeller har en inbyggd grill, alltså ett klassiskt värmeelement typ brödrost.

Länk 1 och Microwave_oven innehåller mer än vad du någonsin vill veta om mikrovågsugnar.!

Låt mig passa på tillfället att önska alla läsare av frågelådan en riktigt god jul och ett gott 2009!

Peter Ekström



/Peter E

Nyckelord: mikrovågsugn [25];

1 http://www.elon.se/koprad_mikro.asp

*

Kraft-Rörelse [16040]

Fråga:
Hej Peter. Angående fråga 16001:
När vattnet passerar ut genom klyvöppningnarna uppstår en brist som yttrar sig som ett undertryck, ett ”sug” i bladets xylem(d.v.s de kärl som transporterar vatten i trädet). En stor trädkrona med hundratusentals blad/barr utvecklar ett mycket stort undertryck, ca. -2 MPa eller -20 atmosfärer. Vatten rör sig från rötterna där trycket är högt till bladen där trycket är lågt. Trycket sjunker därvid också i rötterna och vatten kan i sin tur röra sig från marken till rötterna. Vattentransporten är alltså en fysikalisk process som drivs av tryckskillnader. Kapillärkrafter är inte verksamma och osmos är inte verksamt annat än i samband med reparation av xylemet (sker på våren).
/Tobias K, SLU, Umeå

Svar:
Tobias! Du refererar till fråga 16001 .

Jag hade sett artikeln Hur suger träd upp vatten? som du refererar till. Man för ett märkligt resonemang i stycket Osmos får saven att stiga och rensar rören:

"Genom osmos följer vatten med in i xylemet och en vattenpelare stiger i xylemet som sveper med sig eventuella ”luftproppar” på vägen. ”Luftpropparna” måste avlägsnas eftersom de förhindrar vattentransport genom undertryck. Utsätter man luft för undertryck kan man i bästa fall skapa vakuum (0 MPa) men inte undertryck. En vattenpelare kan däremot sättas under undertryck eftersom vattenmolekylerna hålls samman sinsemellan och med cellväggarnas cellulosa med mycket starka krafter. Kraften är s.k. vätebindning mellan negativt laddade syregrupper och positivt laddade vätegrupper på molekylerna. Motsvarande bindningar finns inte mellan molekyler i gasfas. Om luft sugs in i en vattenpelare under undertryck förloras undertrycket; för att återskapa undertrycket måste luften avlägsnas vilket alltså kan ske m.h.a. osmos hos vissa arter."

Det är flera märkligheter i detta stycke.

* Man säger faktiskt att vattenpelaren byggs upp genom osmos. Är det inte så då att detta övertryck bevaras även i fortsättningen?

* Man säger också, helt korrekt, att vakuum bara möjliggör ett undertryck på 1 atmosfär.

* Ja, vätebryggorna håller ihop vattenmolekylerna men de kan inte lyfta en vattenpelare. Det är som en katt som lyfter sig själv i svansen!

* Vätebryggorna ger en kraft mot cellväggarna, men detta är ju kapillärkraften som påstås vara försumbar!

Det enda jag kan säga med absolut säkerhet är att det är fysikaliskt omöjligt att skapa ett undertryck på 20 atmosfärer om man bara har atmosfärstryck. Undertrycket är max 1 atmosfär, eftersom vakuum är vakuum och du kommer helt enkelt inte längre än trycket 0. Om du kan bygga upp ett övertryck i rötterna på 20 atmosfärer, så är allt OK, och det var i princip det jag föreslog.

Man kan om man vill vända på problemet: om du har en vattenpelare med en höjd på 100 m så utövar den ett statiskt tryck längst ner på 10 atmosfärer. Detta kommer du aldrig ifrån!

Du säger i din fråga att "Vattentransporten är alltså en fysikalisk process som drivs av tryckskillnader. Kapillärkrafter är inte verksamma och osmos är inte verksamt annat än i samband med reparation av xylemet (sker på våren)."

Jag håller med om allt detta utom att övertrycket från osmos bara finns på våren. Det måste finnas hela tiden för att bevara den hydrostatiska balansen, se Xylem#Main_function_.E2.80.93_upwards_water_transport .

Nationalencyklopedin säger i sin artikel om osmos: "Ett flertal fysiologiska processer är förbundna med osmos, t.ex. återförandet av vatten till kroppen i njurar och malpighiska rör och upprätthållandet av vätsketrycket (turgor) i växter." Se fråga 15599 för mer om det senare.

Jag tror en del av problemet är en kulturskillnad: fysiker och biologer har olika sätt att beskriva fenomen. En fysiker kan inte acceptera en förklaring som strider mot elementära etablerade fysikaliska lagar. Författaren till artiken ovan är tydligen lite mindre nogräknad vad gäller detta !

Det enda vi egentligen behöver komma överens om att det är ett övertryck nere i roten som kan hålla upp en 100 m vattenpelare. Hur detta övertryck skapas kan man emellertid diskutera. Det borde gå att dels mäta övertrycket i roten och dels koncentrationen av lösta joner/molekyler så att man kan beräkna det osmotiska trycket.

Under länk 1 finns faktiskt den enligt mitt tycke korrekta förklaringen: "This experiment shows us how osmosis works. If we think about our trees again; the water inside the roots has a higher concentration of sugar as the water on the outside. Through the process of osmosis the tree draws water from the soil, where the concentration is lower, to the inside of the roots, where the concentration is higher, and that is how the tree pumps the water from the soil to the leaves at the top."

För att förvirra diskussionen ännu mer påstås det i artiken under länk 2 att det är kapillärkraften som transporterar upp vattnet! Se även fråga 16172 och Water ascent in tall trees: does evolution of land plants rely on a highly metastable state? .

Tillägg 28/7/10:

Artikeln The Water Circuit of the Plants - Do Plants have Hearts? av Wolfgang Kundt och Eva Gruber, visar att diskussionen om vattentransport i höga träd inte är död. Artikeln är inte helt lätt att förstå, men det är klart att ett övertryck i rötterna orsakat av osmos är en viktig del av förklaringen och att "gummibandsteorin" inte håller. Man inför även - utan experimentella bevis - en sorts mekanisk pump. Denna - med en frekvens på 0.5-15 Hz - motsvarar hjärtat hos t.ex. däggdjur. Med tanke på bristen av experimentella bevis för denna pump, måste jag säga att jag är tveksam.

Artikeln är publicerad i ett öppet arkiv (arXiv.org) som används mycket för vissa naturvetenskapliga ämnen, men artikeln har inte utsatts för normal "peer review". Även detta bör ringa en varningsklocka.

Tack till Bengt för att du gav mig länken till artikeln!
/Peter E

Nyckelord: osmos [8]; vetenskaplig metod [18]; *fysiologi [13];

1 http://www.learnerscience.com/documents/blog.php?title=natural-water-pumps-(osmosis%2C-trees)&entry_id=1205247237
2 http://forestry.about.com/od/treephysiology/p/tree_water.htm

*

Elektricitet-Magnetism [16078]

Fråga:
Magnetkamera
/Veckans fråga

Ursprunglig fråga:
Hej, jag går inte alls i skolan, med frågar ändå. Igår på "Dr House" så blev en patient skadad under en "magnetröntgen" eftersom han hade svalt en nyckel som slets igenom en del organ av magneten. Hur stor kraft kan det röra sig om? Vi kan väl anta att en nyckel väger 20g och att den var tillverkad av vanligt järn.
/Erik S, Stockholm

Svar:
Erik! Det viktiga är att frågan är av allmänt intresse och med åtminstone en anknytning till fysik. Din fråga uppfyller detta väl; det är säkert många som sett Dr House och undrat: är det sant eller påhittat?

Ja, det mycket starka magnetfältet (1-2 tesla) i en magnetkamera utgör ett klart säkerhetsproblem. För det första skulle skiftnycklar, saxar mm från omgivningen kunna slungas med stor kraft mot magneten och därmed patienten. För det andra måste man innan man börjar en undersökning se till att inget ferromagnetiskt finns i patienten. Detta skulle kunna vara allt från proteser, pace-makers etc. till små järnpartiklar som kan finnas i ögat om man sysslat med svetsning. Se mer under MRI#Safety .

Länk 1 innehåller ev Video producerad av en tillverkare av magnetkameror. Där behandlas bland annat faran med magnetiska material i magnetkameror. Där visas bland annat att en vanlig skiftnyckel kan få tillräcklig hastighet för att slå sönder en tegelsten! Så din nyckel i magen skulle säkert kunna göra stor skada!

Vad är då magnetröntgen och vad används det till?
(delar av det följande kommer från svenska Wikipedia)

MRI (Magnetic Resonance Imaging, tidigare kallat Nuclear Magnetic Resonance {NMR på svenska kärnspinnsresonans} men numera omdöpt utan det i vissa sammanhang negativa 'Nuclear') är en metod att avbilda inre organ ofta som ett komplement till röntgenundersökningar. I stället för att som röntgen vara bra på avbildning av t.ex. ben (tyngre ämnen som calcium absorberar röntgenstrålning) så avbildar man väteförekomsten med MRI. Det betyder att metoden är utmärkt för av avbilda även mjuka delar av kroppen, t.ex. hjärnan.

Magnetisk resonanstomografi (MRT) eller Magnetic resonance imaging (MRI) är alltså en medicinsk teknik för bildgivande diagnostik med en magnetkamera (MR-kamera). Tekniken används för att i undersökta patienter upptäcka, lägesbestämma och klassificera vissa sjukdomar och skador som är dolda eller svåra att se vid röntgen- eller datortomografiundersökning.

Magnetkameran bygger på fenomenet kärnmagnetisk resonans som har varit känt sedan 1940-talet. Tekniken bakom den medicinska bildgivande tekniken utvecklades dock i först i början av 1970-talet av bland andra kemisten Paul Lauterbur och fysikern Sir Peter Mansfield vilka belönades för detta med nobelpriset i fysiologi eller medicin år 2003, se länk 2. Användningen av magnetkameror inom sjukvården började på 1980-talet.

Magnetkameran består av stor statisk elektromagnet i form av en tunnel i vilken patienten läggs. Till det statiska magnetfältet kan varierande fält från flera mindre spolar genereras. Ytterligare spolar fungerar som sändare respektive mottagare av radiovågor.

Magnetresonanstomografi (MRT) använder väteatomkärnor, eftersom väteatomen är den vanligaste i människokroppen. Dessa (som ju har spinn och ett magnetiskt moment) riktar sig mot eller med fältet av det statiska magnetfältet. Tillstånden med vätekärnans magnetiska moment i samma riktning och i motsatt riktning i förhållande till magnetskamerans fält har olika energi, och ett radiofrekvent fält kan inducera övergångar mellan tillstånden. Varje gång radiovågorna slås av, återgår atomerna till sitt ursprungliga läge, samtidigt som de avger nya radiovågor. Dessa fångas upp av en antenn och informationen omvandlas av en dator till detaljrika tvärsnittsbilder av kroppens inre, se nedanstående animering av ett människohuvud. Genom att låta magnetfältet variera i olika delar av objektet kan man få fram bilder på ett plan i taget. Data kan sedan behandlas i en dator för att producera en serie skivor som i nedanstående bild (från Wikimedia Commons). För vissa tillämpningar kan man producera en komplett 3D-bild som kan roteras.

Mer att läsa om MRI: länk 2 och Magnetic_resonance_imaging .



/Peter E

Nyckelord: MRI [5];

1 http://video.google.co.uk/videoplay?docid=-3911691380555981564
2 http://nobelprize.org/nobel_prizes/medicine/laureates/2003/

*

Energi [17071]

Fråga:
Hej,

Jag såg en film på youtube där en man ringer in till "ring P1" (2008-12-01).

Man säger att solceller bidrar till växthuseffekten eftersom att de 80% som inte blir energi blir värme. Det låter ju som strunt. Dessutom säger han att något om att den belysta ytan ökar om du har en solcell med en reflektor, jämfört med platt mark. Det låter väl rimligt, men då borde det väl bli mindre energi per ytenhet?

Han blir då bemött med att "den här energin fanns ju där från början". Annars skulle det väl vara nån sorts evighetsmaskin. Men jag kan ändå inte komma på hur man ska tänka för att förklara var resonemanget brister. Kan ni hjälpa mig?
/Peter L, Naturhumanistiska gymnasiet, Malmö

Svar:
Hej Peter!

Ring P1 är en samlingsplats för "nutties", och jag slår genast över till P4 när Ring P1 börjar. Den intellektuella nivån i P4:s frågesportprogram är einstein-nivå jämfört med Ring P1!

Du har helt rätt att uppringaren pratar nonsens. Vad gäller den låga verkningsgraden så är det inget problem eftersom det på rätt plats finns massor av energi i solljuset (se kommentaren om verkningsgrad i fråga 17042 ). Resonemanget att de 80% som inte blir elekticitet ger en extra grobal uppvärmning är inte sant: all solinstrålning på en yta ger uppvämning, se fråga 16846 . Möjligen kan reflektionsförmågan (albedo) hos en solcell vara lite mindre än medelvärdet för jorden. Men ytan som solcellerna upptar är mycket liten. I princip skulle man kunna kompensera ändringen i albedo med vita ytor.

Ett par citat från inslaget: "du ökar aperturarean den belysta ytan med en veckad sofångare" och "om du tar en kvadratmeter och lägger dit en solfångare på två kvadratmeter så blir det dubbelt så varmt".

Det är klart att veckningen inte har någon betydelse. Den maximala effekten man kan få ut är den effekt som motsvarar tvärsnittsytan vinkelrätt mot solen. Detta är ju anledningen att det är kallare i Sverige än vid ekvatorn: solljusets effekt sprids över en större yta. Dessutom är uttrycket "dubbelt så varmt" fullständigt nonsens!

När det gäller storskalig elektricitetsproduktion är solkraftverk med speglar som koncentrerar solljuset, värmer vatten som driver en generator en mycket lovande teknik som redan finns bland annat i Spanien i kommersiell drift, se Solar_power#Concentrating_solar_power och nedanstående bild från Wikimedia Commons.



/Peter E

Nyckelord: solenergi [14]; växthuseffekten [36];

*

Värme [17295]

Fråga:
Inga av de tabeller som jag hittat med hur många watt en lågenergilampa motsvarar, anger värmen. Min fråga är om man tycker att ett rum är för mörkt, och armaturen säger "Max 25W", skulle man kunna sätta en lågenergilampa som motsvarar 40 eller 60w? Är det bara värmeutvecklingen som är begränsningen?
/Peter S, Staffanstorp

Svar:
Peter! Ja, om bara lampan passar hyggligt i fattningen så går det bra. Begränsningen i watt-talet är för den uppvärming som en vanlig glödlampa orsakar. Lågenergilampor (se olika typer i fråga 723 ) har betydligt mindre värmeförluster än traditionella glödlampor. Klarar armaturen en 25W glödlampa, så klarar den en 25W lågenergilampa. Denna motsvarar (se tabellen mot slutet i fråga 723 ) en glödlampa på 100W.

Sedan är det även ett faktum att de ekvivalenta watt-tal som tillverkarna av lågenergilampor ger (denna lågenergilampa motsvarar en X watts glödlampa) är rätt optimistiska. Till en del kan detta bero på att det spektrum som utsänds från en lågenergilampa är "kallare" än det från en glödlampa, så ögat är mindre känsligt för det. (Kallare betyder här mer blått vilket kan vara förvånande om man tänker på temperaturstrålning. Det är emellertid konventionen när det gäller belysning. Jag antar orsaken är att blått ljus ligger långt ifrån infrarött som ju är värmestrålning.)

Observera att för enheten för ljusstyrka, candela (Candela ) ingår känsligheten hos ögat. Ögats effektivitet tas alltså med i beräkningen. Figuren nedan visar känsligheten (figuren från Wikimedia Commons) för ett icke mörkeradapterat öga (svart kurva) och ett mörkeradapterat öga (grön kurva). Den svarta kurvan ger alltså en medelrespons för de färgkänsliga tapparna och den gröna ger responsen för de färgblinda stavarna.



/Peter E

Nyckelord: lågenergilampa [13]; färg/färgseende [39];

*

Energi, Materiens innersta-Atomer-Kärnor [17662]

Fråga:
Kall fusion på italienska
/Veckans fråga

Ursprunglig fråga:
Hej! Jag undrar vad du tror om den senaste rapporten om kall fusion? Vad är nytt jämfört med det som kom upp 1989?
/Arne S

Svar:
Det finns både likheter och skillnader. Likheten är att man presenterar nyheten på en presskonferens och förbigår den normala proceduren att publicera en fullständig rapport i en refereegranskad tidskrift. Den största skillnaden är att man föreslår en helt annan typ av fusion än vätefusion, nämligen fusion mellan väte och nickel. Mer om detta nedan.

Två italienska forskare, Focardi och Rossi, har den 14 januari 2011 demonstrerat en apparat som under en timme levererat en effekt på 12 kW med en input-effekt på 400 W. Hur apparaten är konstruerad säger man inte (av patentskäl), men den innehåller nickel och väte. Se NyTekniks artikel (länk 1) och nedanstående bild på apparaten (från länk 2).

Om funktionen hos apparaten verkligen är vad man hävdar är detta utan tvekan den största uppfinning som mänskligheten någonsin åstadkommit -- den skulle utgöra en i praktiken outsinlig källa till energi. Problemet är att det är ganska säkert inte sant eftersom det skulle kräva en fundamental ändring i fysikens lagar som vi uppfattar dem i dag. Detta är mycket likt Randell Mills idéer (fråga 14237 ) om ett nytt grundtillstånd i väte som kan utnyttjas som energikälla.

Uppgifter om data för demonstrationen är ganska vaga, men vi använder följande som approximationer för nedanstående överslagsberäkningar:

Tid för fortfarighetstillstånd i demonstrationen: 1 timme
Nettoeffekt (Put-Pin): 10 kW
Mängd väte förbrukat: 1 g
Flöde av vatten: 4.9 g/s

Vi antar dessutom i fortsättningen att sedan många decennier etablerade lagar i kärnfysiken gäller.

Artikeln av och Focardi och Rossi

Uppfinningen kallas energikatalysator (Energy Catalyser Boiler), men kunde även kallats evighetsmaskin. Det senare namnet kunde emellertid ställa till problem vid patentansökan .

Artikeln (länk 2) har refuserats i flera tidskrifter. I stället har författarna skapat en ny (webb)tidskrift kallad Journal of Nuclear Physics där ett antal märkliga artiklar publicerats.

Artikeln har flera av de i fråga 14237 listade kriterierna på pseudovetenskap. Mycket i artikeln är korrekta men triviala textboksfakta som t.ex. inledningen om bindningsenergi för atomkärnor och diskussionen om kvantmekanisk tunnling mot slutet.

Det som brister är emellertid beskrivning av apparaten. Det som i normala artiklar kallas experimentella procedurer saknas helt. Detta är helt i strid med det vetenskapliga arbetssättet (fråga 14237 ): alla experimentella resultat skall kunna reproduceras. Vem som helst skall alltså kunna upprepa experimentet och få samma resultat, men detta kan man ju inte göra utan en detaljerad beskrivning.

I diskussionen mot slutet i artikeln förklaras barriärpenetreringen med någon effekt att elektron och proton kommer mycket nära varandra. Elektronen skulle då skärma protonens laddning och protonen skulle därmed inte utsättas för en repulsiv coulombbarriär. Det skulle kanske kunna bli en ny film: Honey, I Shrunk the Hydrogen Atom (jämför Honey,_I_Shrunk_the_Kids ), men någon ny energikälla är det knappast.

Det finns andra märkligheter i artikeln där författarna visar att deras kunskaper i kärnfysik är mycket begränsade. I stället för att i detalj dissikera artikeln och utvärderingsrapporten (som också finns under länk 2) gör vi några enkla beräkningar.

Kemisk energi från väte

Energipotentialen från väte (den energi man maximalt kan få ut av en massenhet väte genom kemiska reaktioner) är enligt figuren i fråga 17516 drygt 140 MJ/kg. Från ett gram väte kan man alltså få ut 140 kJ. Om vi dividerar detta med den utvecklade medeleffekten får vi

(140 kJ)/(10 kJ/s) = 14 sekunder.

1 g väte skulle alltså räcka i 14 sekunder - långt från en timme som demonstrationen varade. Kemiska reaktioner med väte kan alltså inte förklara energiutvecklingen.

Strålning från fusion

Den mest förekommande nickelisotopen är 58Ni. Den dominerande fusionsreaktionen bör då vara:

1H + 58Ni -> 59Cu

Denna reaktion har Q-värdet 3.4 MeV. Det betyder att den exciterade 59Cu kärnan måste på något sätt göra sig av med 3.4 MeV för att hamna i sitt grundtillstånd. Det enda kända processen för detta är genom gammasönderfall. 59Cu är radioaktiv och sönderfaller med halveringstiden 80 s med b+-sönderfall:

59Cu -> 59Ni + e+ + v

Q-värdet för sönderfallet är 4.8 MeV. Totala utvecklade energin per reaktion är då 3.4+4.8=8.2 MeV. Detta värde är helt i linje med den maximala bindningsenergin per nukleon i figuren i fråga 1433 på drygt 8 MeV.

59Ni är mycket långlivat så vi kan bortse från det sönderfallet. Vi bortser även att en del energi försvinner ut ur systemet i form av neutriner.

Antal fusionsreaktioner som krävs per sekund för att generera 10 kW:

(10*103 [J/s])/(8.2 MeV * 1.6*10-13 [J/MeV]) = 0.8*1016 /s

För varje fusion med 58Ni får vi även ett sönderfall av 59Cu. Vi har alltså minst 2*0.8*1016=1.6*1016 gammasönderfall/s.

Aktiviteten blir 1.6*1016/(3.7*1010)=4.3*105 Ci.

Detta är en enorm aktivitet. I Curie sägs att

A radiotherapy machine may have roughly 1000 Ci of a radioisotope such as cesium-137 or cobalt-60. This quantity of nuclear material can produce serious health effects with only a few minutes of exposure.

Den visade apparaten är alldeles för liten för att innehålla tillräckligt med strålskydd. Alla närvarande borde alltså ha fått en dödlig stråldos. Man har dessutom mätt med strålningsdetektorer och konstaterat att strålnivån inte överstiger bakgrundsnivån.

Ett annat problem är att om en stor andel av gammastrålningen slipper ut, så blir det för liten effekt för att skapa apparatens påstådda positiva nettoeffekt.

Strålskydd/infångande av energi

Om fusionen sker med etablerade lagar så kommer energin i huvudsak ut i form av gamma-strålning. Låt oss se hur mycket strålskydd man behöver för att fånga in så mycket strålning att det inte är farligt att gå nära apparaten. Observera att man måste ändå stoppa det mesta av strålningen för att kunna ta tillvara energin.

Halveringstjockleken för bly för gammastrålning med energin 2-4 MeV är c:a 20 g/cm2. Med blys densitet 11 g/cm3 blir halveringstjockleken i cm

20 g/cm2/(11 g/cm3) = 2 cm

Med aktiviteten ovan på 4*105 Ci och en rimlig säker nivå på 1 mCi (som en vanligt lab-preparat) får vi en absorptionsfaktor på

1*10-6/(4*105) = 2.5*10-12

Vi antar vi behöver x halveringstjocklekar:

2-x = 2.5*10-12

(-x)*log10(2) = log10(2.5) -12

(-x)*0.30 = 0.40 -12 = -11.6

x = 11.6/0.3 = 39

Vi behöver alltså ett blyskydd på 39*2 = c:a 80 cm för att få ett säkert strålskydd! Detta hade man uppenbarligen inte vid demonstrationen!

Energi från fusion

Antag att vi har 0.8*1016 fusionsreaktioner per sekund (se ovan). 1 g väte är 1 mol väte. 1 g väte innehåller 6.022*1023 väteatomer. Konstanten är Avogrados tal. 1 g väte räcker då i

6*1023/(0.8*1016) = 7.5*107 s = 21000 timmar. Energipotentialen i 1 g väte vid fusion är alltså mer än tillräcklig.

Demonstrationen av nettoeffekten på 10 kW

Apparaten värmer vatten från 13oC till 100oC (DT=87 K). Enligt fråga 14203 är vattens specifika vämekapacitet 4.18 J/gK. Sedan förångas vattnet (kräver 2260 J/g). Med vattenflödet 4.9 g/s får vi den utvecklade effekten

(4.9 [g/s])*(4.18 [J/(g*K)]*87 K + 2260 [J/g]) = (1.8+11.1)*103 = 12.9 kW

vilket är nära den påstådda effektutvecklingen.

Sammanfattning och slutsats

Om den uppvisade apparaten skulle fungera krävs en fullständig revision av våra kunskaper om atomkärnor. Artikeln är dessutom av pinsamt låg kvalité med många tecken på pseudovetenskap. Att artikeln refuserats i flera tidskrifter visar bara att refereesystemet fungerar bra.

Det finns tre alternativ för den s.k. energikatalysatorn:

  1. Den fungerar som beskrivet och uppfinnarna blir rika som troll (osannolikt)
  2. Man har gjort ett oavsiktligt misstag (knappast, eftersom den påstådda effekten är mycket stor)
  3. Det är ett medvetet bedrägeri för att lura till sig riskkapital (troligaste; detta är numera tyvärr ganska vanligt)

Låt oss fundera på om den utvecklade effekten på 10 kW är rimligt. En normal spisplatta utvecklar c:a 1 kW. När den varit igång en stund är det lätt att känna värmen från plattan om man står inom c:a 1 meter. Effekten 10 kW (som ju måste ta vägen någonstans) borde vara märkbar för alla som var i rummet vid demonstrationen - det borde blivit varmt som i en bastu! Inget sägs emellertid om att det blev varmt i rummet.

Vart tog energin vägen då? Lagrades i apparaten? Nej, det finns inget material med så hög specifik värmekapacitet att det skulle vara möjligt. Försöket är alltså inte enbart tvivelaktigt vad gäller våra kunskaper i kärnfysik. Det verkar även som om apparaten strider mot fysikernas heligaste lag: lagen om energins bevarande.

Man skall inte avslöja trollkarlens trick, men jag har ett förslag till lösning av mysteriet. Det sägs ingenstans, och syns ingenstans i videor och på bilder vad man gör med ångan som genereras. Om denna kondenseras inne i apparaten och släpps ut som vatten återfås ångbildningsvärmet. Enligt ovan används huvuddelen av effekten till att förånga vattnet, så återvinning skulle göra att man kan få en effektbalans utan att blanda in fusion.

Det finns många saker som tyder på att energikatalysatorn inte kan fungera som man hävdar. För mig är de viktigaste problemen avsaknaden av tydlig netto-effektutveckling och avsaknaden av gammastrålning.

Energiutveckling:
Vart tog den utvecklade energin (i första hand i form av vattenånga) vägen under demonstrationen? Kan man verkligen vara säker på att allt kylvatten förångades? Den mesta energin upptas genom ångbildningsvärmet, så det är helt avgörande att visa att allt vatten i form av vattenånga tas ut ur systemet.

Kärnfysikproblem:
I artikeln (länk 1) sägs att det är frågan om fusion mellan nickel och väte till isotoper av koppar. Man hävdar även (utan någon beskrivning av metoden) att man detekterat ett från det naturliga förhållandet avvikande värde på isotopförhållandet för kopparisotoper.

Var och en med elementära kunskaper i kärnfysik kan se ett antal problem med Rossis demonstration och beskrivning.

  1. I tabell 3 i Rossis artikel anges den totala utvecklade energin (Q-värdet) för vätefusion med 58Ni till 41.79 MeV. Detta är totalt felaktigt, det korrekta värdet är 8.2 MeV. Det senare värdet är i god överenskommelse med bindningsenergin per nukleon i detta område, se figur 1 i artikeln.

  2. Coulomb-barriären för en proton mot nickel är av storleksordningen 1 MeV. Transmissionen beräknad med etablerade kvantmekaniska metoder är nästan noll - ett faktum som även Rossi konstaterar i artikeln. Det är svårt att se hur den kemiska miljön skulle väsentligt kunna påverka kärnans Coulomb-barriär.

  3. Även om man accepterar att barriärpenetrationen inte är något problem så är det svårt att se hur den resulterande kärnan överför sin överskottsenergi till omgivningen utan gammastrålning.

  4. Det finns absolut inget skäl att den bildade radioaktiva 59Cu-kärnan skulle sönderfalla på ett helt annorlunda sätt än vad som är väl etablerat. Man borde alltså detektera gammastrålning och annihilationsstrålning.

  5. I fusionen med 58Ni bildas en stor aktivitet av 59Cu (av storleksordningen 1016 Bq, se ovan) som b+-sönderfaller till 59Ni med en halveringstid på 82 sekunder. I artikeln sägs att "No radioactivity has been found also in the Nickel residual from the process". Med tanke på den mycket höga aktivitet som måste ha producerats när energikatalysatorn kördes är detta uttalande ytterst förvånande: det borde vara lätt att verifiera produktionen av 59Cu genom att detektera gammastrålning.

Diskussion
För nästan exakt 100 år sedan (7 mars 1911) presenterade Rutherford sin modell av atomen med en mycket liten kärna (10-15 m) som innehåller nästan hela atomens massa och elektroner som rör sig omkring kärnan inom ett område på c:a 10-10 m. Sedan dess har ett stort antal experimentalister med sofistikerad utrustning och teoretiker med kraftfulla datorer studerat atomkärnans egenskaper. I dag måste man säga att vi förstår atomkärnan mycket väl. Det är svårt att tro att 100 års forskning om atomkärnan är så bristfällig som punkterna ovan indikerar.

Vad gäller tunnlingen skulle man kunna tänka sig att elektroner på något sätt skärmar barriären och släpper in protonen. Problemet med detta är att Heisenbergs obestämdhetsrelation förbjuder elektroner att vistas en längre tid i ett så litet område som atomkärnan. Denna förklaring skulle alltså på ett grundläggande sätt förändra kvantmekaniken som vi känner den.

En annan förklaring av tunnlingen som framförts (Widom-Larsen Theory Portal ) är att protonen växelverkar med en elektron och förvandlas till en neutron och en neutrino (neutronen är ju neutral och har inga problem att ta sig in i kärnan):

p + e- -> n + v

Denna reaktion är fullt tillåten, men problemet är att den sker med den svaga växelverkan och har därmed en mycket liten sannolikhet. Om man vill använda ovanstående reaktion i förklaringen måste man väsentligt modifiera den väl etablerade teorin för den elektrosvaga växelverkan. Dessutom borde man vänta sig att en del av de bildade neutronerna "smiter ut". De skulle då reagera med omgivningen och ge upphov till lätt detekterbar gammastrålning.

I punkterna 3, 4 och 5 ovan är problemet att man i Rossis försök inte observerar någon gammastrålning. En förklaring som framförts är att gammastrålningen från kärnan på något (magiskt?) sätt förvandlas till värmestrålning. Detta är något som aldrig observerats och det står helt i strid med vad vi vet om elektromagnetisk strålning.

Bakgrundsmätning
Om man verkligen vill visa att energikatalysatorn fungerar borde man göra en blind bakgrundstest: Enligt Rossis artikel är vätet nödvändigt för att apparaten skall producera energi. Låt en oberoende person kontrollera väteflödet utan att konstuktörerna vet om flödet är på eller av. Kontrollera att energiproduktionen är fullständigt korrelerad med vätetillförsel.

Låt oss avsluta med ett citat från Carl Sagan: Extraordinary claims require extraordinary evidence.

Bra sammanfattning av Göran Ericsson, Uppsala universitet: Kall fusion i Italien
En uppdaterad diskussion av Peter Ekström: Kall fusion på italienska
Kjell Alekletts synpunkter och diskussionsforum (på engelska): Rossi energy catalyst – a big hoax or new physics? .



/Peter E

Nyckelord: kall fusion [8]; pseudovetenskap [11]; nyheter [11];

1 http://www.nyteknik.se/nyheter/energi_miljo/energi/article3073394.ece
2 http://fragelada.fysik.org/resurser/Rossi-Focardi_paper.pdf

*

Ljud-Ljus-Vågor [17977]

Fråga:
Jag undrar varför varför himlen är blå. Och kan ni förklara våd våglängd är. Snälla svara så fort som möjligt Behöver hjälp nu.
/Ebba L, Mariapark, Helsingborg

Svar:
Ebba! Vi svarar alltid så snabbt vi kan. Vill du har svar omedelbart finns antagligen svaret redan i databasen! Vi har flera svar, se blå himmel .

Våglängden är avståndet mellan repeterande delar av ett vågmönster, t.ex. avståndet mellan två maxima, se Våglängd .



/Peter E

*

Blandat [18385]

Fråga:
Vilka hälsoproblem kan uppstå när man använder stearinljus eller andra levande ljus i våra hem?
/Bertil K, Olympia, Helsingborg

Svar:
Det är inget att oroa sig för om man bara inte har för många ljus och dålig ventilation. Brinnande ljus skapar aerosoler (små partiklar som svävar i luften) av bland annat oorganiska salter, paraffin och sot. Aerosoler kan fastna i lungorna, och det är klart bevisat att detta orsakar sjukdomar, bland annat lungcancer.

Länk 1 är ett examensarbete (ganska avancerat) som visar att det forskas på området.

I artikeln länk 2 (delvis baserat på examensarbetet ovan) visas att ett ljus som brinner stadigt till största delen ger ifrån sig aerosoler i form av vattenlösliga oorganiska salter från tillsatser i veken. Dessa aerosoler har, eftersom de växer i fuktig omgivning (de är hygroskopiska), mindre sannolikhet att deponeras i lungan. Om en saltaerosol ändå deponeras kan den, eftersom den är vattenlöslig) föras bort från lungan. Detta betyder att aerosoler från stearinljus är betydligt mindre skadliga än icke vattenlösliga och icke hygogoskopiska aerosoler som t.ex. sot.

Det finns alltså vetenskapliga argument för att det inte är så farligt att tända några juleljus!

Låt mig passa på tillfället att önska alla läsare av frågelådan en riktigt god jul och ett gott 2012!

Peter Ekström



/Peter E

Nyckelord: aerosol [4]; stearinljuslåga [16];

1 http://www.cast.lu.se/diploma_projects/Examensarbete_Christina_Isaxon.pdf
2 http://www.lu.se/o.o.i.s?id=12588&postid=1401822

*

Värme [18487]

Fråga:
Frysa vatten med värmepump för uppvärmning
/Veckans fråga

Ursprunglig fråga:
Hej. Jag fick en liten fundering. Vi har en elslinga i köksgolvet hemma. Den ger värme av direktverkande el. Nu undrar jag om jag skulle skulle "vinna energi" om jag istället för att använda energin i elslingan, satte in 2 st 10-litershinkar med vatten i frysen. Energin som ligger lagrat i vattnet borde transporteras ut i rummet och jag kan kasta ut isen på gräsmattan. Håller detta resonemang? Tack. /Peter
/Peter N, Skoghall

Svar:
Kul fråga Peter! Det du föreslår är att använda kylskåpet som en värmepump med vatten direkt från kranen som värmereservoir.

En värmepump är en teknisk anordning som överför värme från en kall till en varm plats (Värmepump ).

Enligt termodynamikens andra huvudsats, se fråga 15733 , kan värme bara gå från varmt till kallt. Man kan emellertid komma ifrån detta om man på ett fiffigt sätt utför ett arbete. Detta är vad ett kylskåp gör: med hjälp av energi från en elektrisk motor "pumpas" värme från en kall reservoir (kyl/frys utrymmet) till en varm reservoir (luften bakom kylskåpet). En värmepump är konstruerad precis som ett kylskåp, enda skillnaden är att det är den varma reservoiren som är den intressanta.

Ditt förslag är alltså att använda hinkar med vatten som din kalla reservoir. I normala fall använder en värmepump t.ex. uteluften eller marken i trädgården som kall reservoir.

Vi kan för att definiera storheter rita en schematisk figur på processen, se nedan. Vi har en kall reservoir (blå), en varm reservoir (röd) och en motor som utför arbete. Q är värmemängder och W är den till motorn tillförda (elektriska) energin.

Den optimala processen är vad som kallas Carnot-processen, så vi antar vi har en förlusfri sådan.

Värmefaktorn (COP, Coefficient Of Performance) för en värmepump definieras som

COP(värmepump) = (det vi vill ha)/kostnaden = QH/W

På analogt sätt kan vi definiera kylfaktorn för en kylmaskin

COP(kylmaskin) = (det vi vill ha)/kostnaden = QC/W

Men totala energin bevaras så

QH = QC + W

dvs

COP(värmepump) = QH/(QH-QC)

För Carnot-processen gäller (Coefficient_of_performance ) att entropin dQ/T är konstant varav följer

QC/TC = QH/TH

dvs

COP(värmepump) = TH/(TH-TC)

Låt oss för enkelhets skull anta vi utgår från nollgradigt vatten och att den varma reservoiren är 50o. Vi får då värmefaktorn

COP(värmepump) = (50+273)/(50) = 6.46

Smältvärmet för vatten är (fråga 14203 ) 333 kJ/kg. Låt oss räkna på 1 kg (=1 liter) vatten:

QC = 333 kJ

COP = 6.46 = QH/(QH-333)

6.46 QH -333*6.46 = QH

QH = 394 kJ

W = QH - QC = 61 kJ

Så med en insats på 61 kJ (elmotorn) får vi ut 394 kJ! Det låter som trolleri, men är faktiskt sant.

Låt oss kontrollera den totala entropin i varma och kalla reservoirerna

S = QH/TH + (-QC)/TC

S = 394/(50+273) - 333/273 = 0.00

vilket är som det skall vara för en Carnot-process.

Nu kan vi räkna ut om din idé är realistisk. Låt oss anta du behöver 5 kW 50 gradigt vatten för uppvämning. 1 kg vatten räcker då

394[kJ]/5[kJ/s] = 78.8 s

Du behöver alltså ställa in en tiolitershink med vatten ungefär var tionde minut! Och det är med en ideal process, en verklig process skulle ha betydligt mindre COP, i bästa fall 30-50% av den ideala.



/Peter E

Nyckelord: värmepump/kylskåp [8]; termodynamik [17]; verkningsgrad [26];

*

Kraft-Rörelse [18622]

Fråga:
Jag fick till svar att pilen har högst hastighet när den lämnar bågen. Hur kommer det då sig att pilen går in djupare i tavlan när man står på t ex 20 m än när man står på en meter(anslagsenergi?). Jag fick svar av Peter E. Vänligen Jens
/Jens I, Holsby, Vetlanda

Svar:
Hej Jens! Du refererar till fråga 18591 . Det finns även en besläktad fråga om handboll, se 14870 .

Det är fysikaliskt omöjligt att pilens hastighet kan öka om det inte finns en extern kraft (se Newtons första rörelselag, fråga 17040 ). Den externa kraften är snarare luftmotståndet som i stället bromsar upp pilen.

Om pilen verkligen penetrerar tavlan mer på 20 meters avstånd än på 1 meters avstånd måste det bero på något annat än hastigheten. Jag har konsulterat en person (tackar!) som har 30 års erfarenhet av bågskytte, och han hävdar bestämt att det är tvärtom: penetrationen av tavlan minskar med skjutavståndet.

Nedanstående video visar emellertid att pilens flykt är mer komplicerad än man skulle tro. Pilen slingrar sig fram som en orm. Kan det vara denna effekt som ger mindre penetration i tavlan på korta avstånd?

Vad som skulle behövas för att etablera fenomenet är väldokumenterade kvantitativa mätninger.
/Peter E

Nyckelord: *idrottsfysik [42];

*

Materiens innersta-Atomer-Kärnor [18638]

Fråga:
Påverkas elektronens hastighet då den cirkulerar i sin bana kring atomkärnan av temperarturen, speciellt intresserad av fallet då T närmar sig 0 Kelvin. /Peter
/Peter A, Katedralskolan, Linköping

Svar:
Nej. Bundna elektroner påverkas inte av temperaturen så länge den är mindre än 1000 K. Sedan bör du inte föreställa dig att elektronerna cirkulerar i en bana. Se fråga 13733 för en bättre modell.
/Peter E

*

Kraft-Rörelse [18654]

Fråga:
Jag har tidigare frågat om energin i vattenfall (17697).

Men Peter E's svar på fråga 16454 i samma ämne (..."Bär du ner en hink går energin till att värma upp dina skosulor."...) gör mig förbryllad.

Antag att jag bär ett par hantlar, 2x5=10kg, uppför en trappa till en nivåskillnad på tio meter. Ganska jobbigt, jag får göra av med en del energi för att utföra detta och väl däruppe har hantlarnas lägesenergi ökat en del. (Förmodligen har även lägesenergin för min kroppsvikt förändrats, måhända svårare att beräkna med tanke på den energi som krävts för arbetet.) Nåväl, detta kan jag köpa. Rörelse- och värmeenergi har krävts för att bygga upp lägesenergi.

Men om jag nu går ner igen med samma hantlar, så minskar deras lägesenergi till sitt ursprungliga värde. Det är svårare att förstå? Slits skosulorna mera när jag går nedför trapporna? Det tror jag inte. Snarare tvärtom.
/Curt J, Huddinge

Svar:
Du refererar till 16454 och 17697 .

Om du skall kunna återvinna den potentiella energin måste det finnas en process som gör det. Vattnets rörelseenergi omvandlas till rörelse hos turbinen som driver en generator.

Din hanteltransport blir inget annat än lite nyttig motion. All rörelseenergi du vinner när du går nerför trappan förloras i form av friktionsvärme.

Jo, det är jobbigt att gå nerför en trappa också. Speciellt för knäna!
/Peter E

*

Kraft-Rörelse [18660]

Fråga:
Så här är experimentet formulerat: Din uppgift är att bestämma hur stor del av klossens lägesenergi som övergår till rörelseenergi under dess färd nedför planet.

Till din hjälp har du ett lutande plan, en kloss, en linjal och ett stoppur.

Svar: Du kan använda formlerna i fråga 14687 . Som du ser ingår massan m både i uttrycket för lägesenergi och för rörelseenergi. Eftersom frågan är hur stor del av lägesenergin blir rörelseenergi, så kommer m att försvinna. Du behöver alltså inte räkna ut m. /Peter E

Tack för ditt snabba svar! Vill bara göra detta lite extra tydligt, det är inte helt solklart än, om det går bra.

Ok, då har jag

(Potentiel energi=lägesenergi)---> startläget. P=mgh (Kinetisk energi=rörelseenergi)---> "i mål" K=(mv^2)/2

Men om jag då får ett svar som t.ex. kommer att vara P=5m och K=3m så tar m ut varandra och en del av energin går över i friktion

I så fall går 3/5 av energin till rörelse energi (kinetisk energi) eller 60%, är detta korrekt tolkat?! Och resten i form av friktion som luftmotstånd och materialpåverkan? Finns det några fler ställen energin kan vandra? Och om man lägger ihop K+P=Mekanisk energi?

Lite diffust dehär fortfarande :)
/Mathias H, Komvux, GBG

Svar:
Du refererar till fråga 18659 .

Ja, det är korrekt att i ditt exempel blir andelen rörelseenergi 3m/5m = 60%. Resten försvinner som värme genom friktion och luftmotstånd.
/Peter E

*

Kraft-Rörelse [18821]

Fråga:
Hej! Jag ställde denna fråga tidigare i veckan:

Hej, jag undrar hur man kan bevisa följande faktum: Om man kastar en boll rakt upp vertikalt i luften tar det tiden t innan den landar i marken. Vid vändningspunkten dvs den högsta höjden bollen når har halva tiden, t/2, passerat. /mimmi r, NTI skolan, danderyd

Svar: Därför att accelerationen hela tiden är konstant = g (tyngdaccelerationen). Om begynnelsehastigheten är v0 ges hastigheten v som funktion av tiden t:

v = v0 - gt

I vändpunkten är v=0:

0 = v0 - gt ==> t = v0/g

När bollen träffar marken är v=-v0

dvs

-v0 = v0 - gt ==> t = 2v0/g

vilket är dubbla stigtiden. /Peter E

Tack för jättebra svar! Jag har dock en följdfråga!! När man kastar bollen lämnar den ju handen på en höjd ovanför marken. Detta medför ju att när bollen landar på marken är y = - höjden, om man sätter kastnivån som nollnivå. Stämmer resonemanget ovan även i det fallet? Hur ska man tänka då? Borde det inte ta längre tid för bollen på nervägen än på uppvägen då sträckan ner ju blir längre?
/mimmi r, NTI skolan, danderyd

Svar:
Det är enklast att dela upp rörelsen i två delar: från handen till vändpunkten och från vändpunkten till marken. Alla formler du behöver finns i fråga 18438 .
/Peter E

*

Blandat [18975]

Fråga:
Hej hur ska jag definiera och skilja på balans och jämvikt?
/Peter W, Skogsback, Botkyrka

Svar:
Peter! Det är mycket liten skillnad. Jämvikt är möjligen lite mer tekniskt och generellt.
/Peter E

*

Kraft-Rörelse [19239]

Fråga:
Visa fråga/svar

Kraft-Rörelse [19237]

Fråga: Hej Frågelådan! Letar o letar men hittar inga tidigare svar... Nu frågar jag er o hoppas på svar ;)

G-krafts formel o beräkning vid frontalkrock(bil). 160 km/h och en stoppsträcka på 1 meter.

Vad blir G-kraften?

Vänligen Magnus /Magnus A, Polhem, Lund

Svar: Det finns flera exempel med nyckelordet g-krafter , t.ex. fråga 10076 vilken är mycket lik din fråga. /Peter E

Hej igen o tack för snabbt svar! Blir då beräkningen av G-kraften. 44,44/0,025=1 777,77 G? Med 1,11 m deformationszon? Vänligen Magnus
/Magnus A, Polhem, Lund

Svar:
Nja, inte särskilt rimligt!

v = 160 km/t = 160/3.6 = 44.4 m/s

Uppbromsningstid = t = s/(v/2) = (1 m)/(44.4/2 m/s) = 0.0450 s

Acceleration = a = v/t = 44.4/0.0450 = 987 m/s2 = 987/g g = 100 g
/Peter E

Se även fråga 19237

*

Elektricitet-Magnetism [19707]

Fråga:
Hej! Läser fysik 1 och hade labb igår och skulle behöva hjälp med en diskussionsfråga efter till labbrapporten. Här kommer uppgiften:

Uppgift:Att undersöka sambandet mellan ström och spänning för en resistor och en glödlampa. Materiel: Spänningsaggregat(kub), okänt motstånd(ca 500 Ohm), glödlampa(24 V, 50 mA),2 universal¬instrument och sladdar. Utförande: Kontrollera att kuben är inställd på likström och att instrumenten är nollställda och inställda på lämpliga mätområden. a)Koppla först upp en krets med resistorn där strömmen i kretsen och spänning över resistorn kan mätas. Variera strömmen i kretsen och gör 10-15 mätningar av strömmen och spänningen (i intervallet 0-30 V). OBS! Byt inte mätområde på amperemetern i mätserien! b)Repetera ovanstående mätningar med en glödlampa i kretsen i stället för resistorn. Glödlampan kan överbelastas under kort tid.

Som svar fick jag att med resistorn blev det en konstant ökning, eftersom motståndet är samma hela tiden (?), men vad jag undrar är varför är det INTE en konstant ökning med lampan?

Tacksam för svar! :) //Jennie
/Jennie H

Svar:
Se fråga 4730 .
/Peter E

*

Elektricitet-Magnetism [19827]

Fråga:
Hur kan man magnetisera en stålstav?
/Veckans fråga

Ursprunglig fråga:
Hur kan man magnetisera en stålstav?

fråga är ställd av en elev i mitt namn.
/Diana G, Eriksdalsskolan, Stockholm

Svar:
I länk 1 beskrivs tre sätt att göra en stålstav magnetisk: gnida staven med en permanentmagnget, med spole och batteri och med mekanisk påverkan i det jordmagnetiska fältet. Alla dessa metoder ger ganska svaga magneter.

Det lättaste är nog den första metoden. Man behöver bara en permanentmagnet (så stark som möjligt).

1 Testa att din stålstav påverkas av magneten. Om den inte gör det går den inte att magnetisera.

2 Håll den ena polen (t.ex. S) hos permanentmagneten mitt på stålstaven och dra den utåt kortändan (se nedanstående bild från WikiHow, länk 1).

3 Vänd stålstaven och upprepa processen med den andra polen (N).

Repetera punkt 2 och 3 ett antal gånger.

Denna procedur bör fungera om man vill magnetisera t.ex. en skruvmejsel så att skruvarna fastnar på den. Magnetiseringen förstörs av stötar, så man kan behöva upprepa processen då och då.

För att tillverka starka permanentmagneter krävs lite mer. Den ferromagnetiska metallen upphettas till över curietemperaturen (vid vilken metallens ferromagnetiska egenskaper försvinner, se Curietemperaturen ). Metallbiten utsätts för ett magnetfält (så starkt som möjligt). Temperaturen sänks till under curietemperaturen varvid de upplinjerade magnetiska domänerna fryses.

Se även Magnet#Magnetizing_ferromagnets .



/Peter E

Nyckelord: magnetism [52];

1 http://www.wikihow.com/Magnetize-Steel

*

[19869]

Fråga:
Hej Frågelådan.

Jag såg frågan om "Varför är inte himlen violett då?" Bra fråga, och ett bra svar. Tack för det!

Men, jag undrar om ni inte gör det mer komplicerat än det är.

Violett ljus, som jag ser det, är en blandning av ljus med våglängder i det blåa och röda våglängdsområdet. Det ljus som är i änden av regnbågen, och som man också kan se på himlen, är BLÅTT.

Varför man ofta säger att violett ska finnas i regnbågen, eller varför man kallar strålningen utanför synliga spektrat för ultraviolett istället för ultrablått är ett mysterium för mig, men det har nog med någon sorts språkglidning att göra.

Så den blåa färg som är i änden av synliga spektrumet, den kan man nog se på himlen ibland, iaf på en fin högsommardag.

Se mer på temat på: https://www.youtube.com/watch?v=9udYi7exojk

Hälsningar Peter
/Peter A, Uppsala Universitet, Uppsala

Svar:
Violett ljus är våglängder mellan blått och ultraviolett. Ögat kan emellertid på grund av sin konstruktion med tre olika sensorer (tappar) ge intrycket av en annan färg genom blandning av grundfärgerna RGB (rött, grönt, blått).

Himlen är blå eftersom kortvågigt ljus sprids mer är långvågigt (se fråga 13750 ). Tre faktorer gör att det kortvågiga violett inte dominerar hos det spridda ljuset: Violett absorberas mer än blått i atmosfären, ögat är mindre känsligt för violett (se figuren i fråga 13824 ) och det kommer mindre violett än blått ljus från solen.
/Peter E

Nyckelord: blå himmel [12];

*

Kraft-Rörelse [20093]

Fråga:
Hej Peter,

Vad jag förstått på fysik sajterna IOP Science Physics Education och Physics Stack Exchange så kan allmänna relativitetsteorin tolkas så att motriktade starka ljusflöden typ lasrar attraherar varandra "gravitionellt/rumtidsmässigt?" emedan ljusflöden som har samma riktnign inte påverkar varandra. Går inte detta fenomen att se ute i rymden med starka teleskop eller är effekten för svag?

Mvh Mikael Hjelm
/Mikael H

Svar:
Mikael! Ingen aning - det låter i så fall som en mycket liten effekt. Att ljus böjs av massa (fråga 16021 ), att fotoner har massa (observera massa, inte vilomassa, se fråga 16989 ) och att man får en förskjutning i våglängd i gravitationsfält (fråga 16989 ) är emellertid väl etablerat.
/Peter E

1 http://iopscience.iop.org/article/10.1088/1367-2630/18/2/023009/pdf

*

Elektricitet-Magnetism [20218]

Fråga:
Hej! Under en laboration mätte vi hur stor den inducerade strömstyrkan i en spole vid induktion m hjälp av amperemeter, stavmagnet och spole med järnkärna. En större strömstyrka uppmättes i spolen med 300 varv jämfört med spolen med 600 varv. Beror det på en större resistans i 600 varvs-spolen? Hur förklarar man för årskurs 8-elever vilken spole som inducerar högst spänning? Tack på förhand! Peter
/Peter O, Malmö Idrottsgrundskola, Malmö

Svar:
Jag förstår inte hur du kan sticka in en magnet i en spole med järnkärna.

Den inducerade spänningen är proportionell mot ändringen i magnetfältet och antalet varv i spolen. Strömmen beror av totala resistansen (R, spole och resten av kretsen) genom Ohms lag I =U/R. Det är alltså bättre att mäta spänningen med ett oscilloskop (se länk 1) eller en multimeter.

Att spänningen beror av ändringen i magnetfältet är helt enkelt ett experimentellt resultat (Faradays induktionslag, Faradays_lag ). Proportionaliteten mot antalet varv i spolen beror på att varje varv ger upphov till en spänning dU, så N varv ger en spänning på U=N*dU.
/Peter E

Nyckelord: induktion [13];

1 https://www.kth.se/social/upload/52532286f276545f5583decf/Lab 4.docx

*

[20359]

Fråga:
Hej. Försöker för mitt eget höga nöje repetera lite termodynamik och har kört fast på en fråga som jag egentligen tycker att jag borde klara av. Förkortar frågeställning något: I en hög cylinder löper en kolv av massa 3.5 kg friktionslöst. Kolvens radie är 8 cm. I cylinderns botten finns 2 kg vatten som är strax under 100 °C samt en elektrisk värmare som tillför vattnet värme med effekten 100 W. Precis när vattnet börjar koka, med vilken hastighet stiger kolven (om vi modellerar ångan som en ideal gas)? När sedan allt vatten precis blivit ånga och värmaren överför värme till ångan med samma effekt (100 W), hur snabbt stiger då kolven?
/Urban N, LTU, Luleå

Svaret kommer snart...

*

Ljud-Ljus-Vågor [20511]

Fråga:
Erat svar angående ljusgenomsläpp vad gäller polaroidfilter var följande:

Låt oss först beskriva vad ett polarisationsfilter gör:

Ett sådant filter släpper igenom den komponent som är parallell med filtrets polarisationsriktning. Detta innebär att om dessa riktningar är parallella så släpps allt ljus igenom. Om de är vinkelräta inget och om vinkeln är 45 grader hälften av ljuset. Det ljus som kommer igenom har samma polarisations riktning som filtrets riktning.

Om vi skickar vanligt ljus, som innehåller alla möjliga polarisationsriktningar, mot ett filter så kommer filtret att göra att vi efter filtret har planpolariserat ljus.

Lägger vi två plattor med vinkelräta riktningar efter varandra kommer inget ljus igenom. Lägger vi däremot in ett annat filter mellan med 45 graders vinkel så kommer visserligen detta mellanfilter att absorbera en del ljus men det kommer också att vrida polarisationen så att en del av ljuset kommer igenom det sista filtret. /Gunnar O

Svaret är att om tre filter efter varandra vertikalt sedan diagonalt och sedan horisontellt innebär att fullt ljusgenomsläpp sker efter tredje filtret. Alltså har ni ett stort problem om ni försöker förklara fullt genomsläpp med att mellanfiltret "vrider ljuset " så att fullt genomsläpp sker efter tredje filtret. Det första filtret absorberar hälften hur kan då fullt ljus gå igenom efter tredje filtret oavsett om det är ett diagonalt som andra filter som "vrider" ljuset. Med vänlig hälsning Peter Obs har kollat detta, experimentellt att det stämmer.
/Peter J, Göteborg

Svar:
Du refererar till fråga 12347 .

Där sägs att

...det kommer också att vrida polarisationen så att en del av ljuset kommer igenom det sista filtret...

Det står ingenstans att allt ljus kommer ut efter det tredje filtret. I själva verket kommer 1/2 av intensiteten ut ur det första, 1/4 ut ur det andra och 1/8 ut ur det tredje. Svaret är alltså korrekt men beskrivningen lite kortfattad. I fråga 5528 finns en mer detaljerad genomgång av fenomenet.
/Peter E

Nyckelord: polarisation [7];

*

[20648]

Fråga:
Hej! Undrar om någon vet om man kan köpa en volframkub (1 kg) i Sverige, eller åtminstone inom EU. Har hittat en på en amerikansk sajt men då blir det väl dyr frakt och ev tull. Skulle vara väldigt kul att ha när vi läser om densitet. MVH Peter
/Peter J, Flahultskolan, Norrahammar

Svaret kommer snart...

*

Kraft-Rörelse [20729]

Fråga:
Hej,

jag går i sjuan och undrar om det finns en matematisk formel för evighetsmaskinen "the overbalanced wheel"? På skissen förstår jag ju att denna inte kan fungera med alla långa pendlar med vikten längst ut. Men om man tar mycket kortare pendlar så det inte blir många fler på vänster sida kanske det fungerar? Men jag skulle vilja göra en matematisk beräkning på det innan jag börjar bygga. Hur räknar jag ut varje pendels kraft? För om summan av alla pendlar på högra sidan blir större än vänstra sidans bör den ju snurra?

Hoppas på hjälp?

Lars
/Lars N, Victoriaskolan, Göteborg

Svar:
Hej Lars!

Nej, den kommer inte att fungera även om du kortar pendlarna eftersom den drivande kraften minskar lika mycket som den bromsande. Bilden i fråga 15733 innehåller en förklaring till varför Det Överbalanserade Hjulet inte fungerar som en evighetsmaskin.

Bilden nedan från Perpetual_motion#Techniques visar problemet tydligt: Hjulet är tänkt att rotera medurs. Det finns då 4 drivande, 7 bromsande pendlar (plus 1 neutral pendel).

Evighetsmaskiner är cykliska (repeterande) och verkliga system har friktion. Så en evighetsmaskin strider mot principen om bevarandet av den totala energin (friktionskrafterna utför ett arbete). Man behöver inte visa bristen på funktionalitet med krafter - maskinerna är ofta mycket komplexa - utan energiprincipen räcker.

Man kan åstadkomma system som nästan saknar friktion - t.ex. en roterande leviterad (se fråga 16775 ) magnet i vakuum. Ett annat exempel är en roterande rymdsond som under fritt fall i rymden fortsätter att rotera obegränsad tid. Men dessa exempel kan knappast kallas evighetsmaskiner, eftersom de inte kan ge något användbart arbete.



/Peter E

Nyckelord: evighetsmaskin [14];

1 https://www.bogleheads.org/forum/viewtopic.php?t=80680
2 https://permies.com/t/55559/overbalanced-wheel

*

Ljud-Ljus-Vågor [20753]

Fråga:
Smälla en uppblåst ballong med en blixt
/Veckans fråga

Ursprunglig fråga:
hej, min pappa sa att detta med om man ritar med en svart kvadrat på armen och lysa med blixt på, och att det skulle sticka bara var tro och skrock, jag testade på honom och han erkände faktiskt att det stickte till lite, helt plötsligt kom det bara upp en annan fråga i huvudet, om man ritar en svart fläck på en ballong som inte är svart, kommer det hända något då? isåfall kommer det vara samma anledning som om man gör det på kroppen? Annars är jag väldigt tacksam om du förklarar för mig, min pappa trodde inte på detta heller men han skulle åka och hämta min syrra i skolan oxh jag tror jag lyckades övertala honom om att köpa ballonger så jag kunde testa. //Julia
/Julia p, Lovisedalsskolan, Vallenrtuna

Svar:
Hej Julia!

Kvadraten på armen och en blixt finns utredd i fråga 20427 . Att effekten är verklig är helt klart även om man inte vet exakt vad som händer. Att det har att göra med snabb uppvärmning genom att ljus från blixten absorberas är fullt klart.

Jag tror inte att ballongen påverkas i ditt försök, men det enda säkra är att göra experimentet. Hoppas du kan få tag på en ballong så du kan göra experimentet!

Uppdatering från Julia: "Min mamma köpte hem ballonger och det funkade faktiskt!"

Jag får böja mig för resultatet från ditt experiment! Tack Julia! /Peter

Den högre absorptionsförmågan på det svarta området räcker tydligen för att värma upp ett område på ballongytan tillräckligt för att ballongen skall spricka.
/Peter E

Nyckelord: *vardagsfysik [64];

*

Kraft-Rörelse [20983]

Fråga:
Hej,

Skulle behöva veta hur många watt jag behöver på en vattenpump.

Har ett litet aqaponic-projekt på gång. 1.000 liter vatten behöver lyftas 1,5 meter (ungefär, men det existerar inte i fysiken) i timmen. Är lite "lost" då det var några år sedan jag läste fysik.

Provade med P=mgh/t m=1.000 liter g=9,82 h=1,5m t=60*60=3600s (1timme)

P=(1.000*9,82*1,5) / 3600 = 4W Men är det rimligt? Visst, lite förluster med verkningsgrad och friktion i rören på det, så låt oss dra till med att en vattenpump på 10W. Skulle den verkligen fixa det jobbet?

Nu är väl m massa och enheten då g (gram) men då blir det mer orimligt. P=(1.000.000g*9,82*1,5) / 3600 = 4kW Det är så långt från verkligheten man kan komma hahaha.

Tacksam för svar Peter
/Peter N, Kalasin/Thailand

Svar:
Ditt uttryck är helt korrekt:

P = (m/t)*g*h

Kontroll av enheter (använd alltid SI-enheter!)

[kg/s]*[m/s2]*[m] = [kg*m/s2]/[s] = [J/s] = W

Vi får alltså

P = (1000/3600)*9.82*1.5 = 4 W

Det är en undre gräns eftersom du måste ta hänsyn till att verkningsgraden inte är 100%.

För det omvända problemet vattenkraft (el från strömmande vatten) är verkningsgraden c:a 90%, se fråga 794 .
/Peter E

*

Materiens innersta-Atomer-Kärnor [21003]

Fråga:
Hej Peter,

Tack för svaren. Jag tänkte inte på våg partikeldualismen:) Fick syn på denna fråga och förstår nu lite mer.

Wave particle duality and gravity https://physics.stackexchange.com/questions/450263/wave-particle-duality-and-gravity

Bra tips? Physics Forums Math and Science FAQs

Reference https://www.physicsforums.com/insights/science-math-faqs/
/Mikael H, Västerås

Svaret kommer snart...
/Peter E

1 https://physics.stackexchange.com/questions/450263/wave-particle-duality-and-gravity
2 https://www.physicsforums.com/insights/science-math-faqs/

*

Kraft-Rörelse [21193]

Fråga:
Hej Peter, Tack för senaste svar. Jag har lite frågor kring några labbar jag gjort nu under fysik 1.

Labb 1

Gick ut på att undersöka energiomvandling hos olika händelser. En av de var en curlingsten som glider på blank is med konstant fart. Här sker inga energiomvandlingar. Varför? Jag tänker att acceleration=0 då farten är konstant. Den drivande kraften är alltså lika stor som friktionskraften. Kraftresultanten är 0, stenen är i jämvikt. Kan man anta att friktion inte finns? Stenen lägesenergi är konstant på horisontalplanet men borde inte stenen ha rörelseenergi som omvandlas till friktion, för stenen borde väl stanna någon gång, eller?

En annan händelse var om fler tar trappan istället för rulltrappan kommer då affären att få lägre elräkning? Jag tänker att ju fler personer som står på rulltrappan desto mer kraft måste dess motor prestera för att ta personerna upp till övervåningen.

Lab 2

Undersökning av rörelsemängdens bevarande. En lättare vagn krockar med en tyngre stillastående vagn. Utan att ha avrundat några värden är den totala rörelsemängden före kollision och efter 0,2561886051kgm/s≠0,2564570616kgm/s. Varför denna skillnad? Så här skrev jag Vid kollision med den tunga vagnen kommer en del av den lätta vagnens rörelseenergi att överföras till den tyngre. Detta sker under en process som kallas för deformation. Under deformationen avges även värmeenergi och ljudenergi. Detta gör att systemet inte kan betraktas som slutet vilket gör att vi får variation i total rörelsemängd före och efter kollision. Stämmer det? Vilka mer tänkbara felkällor kan finnas med?

Lab 3 Bestämma 1,5V-batteri ems och inre resistans. I experimentet ökar polspänningen när strömmen minskar samt om strömmen ökar minskar polspänningen. Beror det på inre resistansen? Varför får man inte ut hela polspänningen? När det gäller felkällor i labben tänker jag bla inre resistans hos volt- och amperemetern samt yttreresistans i sladden. Finns det fler felkällor? Skulle man kunna förbättra labben på något vis? Har undersökningen och resultaten någon allmännytta?

Tack på för hand!


/Kristina S, Komvux

Svar:
Det var länge sen sist . 2008!

1 Om stenen har konstant hastighet påverkas den inte av någon netto-kraft, se Newtons 2 lag i fråga 12834 . Utan friktion sker alltså ingen energiomvandling. Om det finns en drivande kraft och farten är konstant finns det en friktionskraft. Denna omvandlar som du säger rörelseenergi till värmeenergi.

Ja, att transportera upp personer kostar energi och denna kommer från att elmotorn får jobba lite mer.
/Peter E

*

[21339]

Fråga:
Hej Peter!

Jag behöver hjälp med två frågor, men mina frågor är på engelska hoppas det är ok. 1) Produce a spectrum of light by immersing a mirror in a vessel of water and allowing the sun's rays to be reflected in the mirror. Why does sunlight divide into different colors? What is the function of water? Where in the spectrum do we find blue light and red light respectively?

2)Cut a cut in a piece of cardboard (eg a playing card) with a razor blade or a carpet knife. Let light from a lamp pass through the section and describe the interference pattern. You can vary the size of the opening by gently bending the piece of cardboard. Make another incision close to the previous incision and repeat the experiment. Describe your observation.

Tack på förhand!
/Per S, Liljeholmens gymnasium, Stockholm

Svar:
Svaret (och frågan) finns under länkarna. Du har bara klippt och klistrat!
/Peter E

1 https://www.chegg.com/homework-help/questions-and-answers/experimental-task-1-produce-spectrum-light-immersing-mirror-vessel-water-allowing-sun-s-ra-q66921286
2 https://www.chegg.com/homework-help/questions-and-answers/experimental-task-2-cut-cut-piece-cardboard-eg-playing-card-razor-blade-carpet-knife-let-l-q66921370

*

[21348]

Fråga:
- Vad är ström?

I vilken enhet mäts ström?

- Vad är spänning? - Den drivande kraft som får elektroner att röra sig.

- I vilken enhet mäts spänning?

- Hur hög är spänningen i vägguttagen?

- Hur hög är spänningen i ett vanligt 1,5-V-batteri?

- Vad är en krets?

- När är den sluten? När är den bruten?

- Vad är en ledare?

- Ge exempel på några material som är god ledare?

- Vilka egenskaper ska en god ledare ha?

- Vad är en isolator?

- Ge exempel på några material som är isolatorer.

- Vad är resistans?

- I vilken enhet mäts resistans?

- Varför använder man sig av motstånd som en komponent i en krets?

- Hur ser symbolen för ledare, batteri, lampa, strömbrytare, amperemätare och voltmätare ut?

- Vilka för och nackdelar finns det med att seriekoppla lampor?

- Vilka för och nackdelar finns det med att seriekoppla batterier?

- Vilka för och nackdelar finns det med att parallellkoppla lampor?

- Vilka för och nackdelar finns det med att parallellkoppla batterier?

Tack så mycket Peter!!!!!!!!!
/Olivia L, Kungshlmens grundskola, Stockholm

Svaret kommer snart...

*

[21361]

Fråga:
jag är så himla tacksam med att du svarar på mina frågor så fort som det går... TACK o(〃^▽^〃)o Du behöver verkligen inte svara på min senaste fråga eftersom jag har redan fått svar på de frågorna :)

Men tack så hemskt mycket Peter!!!!!
/Olivia L, Kungshlmens grundskola, Stockholm

Svaret kommer snart...

*

Sök efter    

Skriv de ord du vill söka på i sökfältet ovan och klicka på sökknappen. Uteslut ord genom att sätta - (minus) före ordet. Ordgrupper definieras med hjälp av "...". Sökningar är oberoende av stora och små bokstäver.

Exempel:

helium "kalle anka"
Sök på 'helium' och ordgruppen 'kalle anka'
orgelpipa
Sök på 'orgelpipa'
orgel -gitarr
Sök på 'orgel' men inte 'gitarr'

 


sök | söktips | Veckans fråga | alla 'Veckans fråga' | ämnen | dokumentation | ställ en fråga
till diskussionsfora

 

Creative Commons License

Denna sida från NRCF är licensierad under Creative Commons:
Erkännande-Ickekommersiell-Inga bearbetningar
.